5 Chapter Five: Property Crimes

Sections in Chapter 5

Introduction
Common Law to Consolidation: Larceny and Beyond
Burglary and Trespass
Robbery
Arson
End of Chapter Review

Introduction

The first four chapters of this book have offered an overview of how criminal law works. They have examined a human practice in which written texts are used to express, guide, authorize, or constrain decisions to classify and condemn certain acts, and certain people, as criminal. Our coverage of the basic structure and operation of criminal law has not, so far, focused on any single type of criminal offense. This chapter and the next two do focus on specific categories of offenses. Before discussing property crimes and the reasons to study them, it may help to take note of the criminal offenses that were charged in the cases you’ve read so far:

  • Commonwealth v. Mochan: misdemeanor injury to public morality (harassing telephone calls)
  • Commonwealth v. Copenhaver: driving while intoxicated and drug offenses (after traffic stop for expired registration)
  • Morissette v. United States: knowing conversion of government property (a kind of theft)
  • State v. Alvarado: possessing contraband in prison
  • Lambert v. California: failure to register as felon
  • People v. Kellogg: public intoxication
  • City of Chicago v. Morales: gang loitering
  • Inmates of Attica v. Rockefeller: [no charges; this was a civil lawsuit seeking to compel prosecutors to charge prison officials with assault, homicide, and civil rights violations]
  • State v. Cissell: failure to pay child support
  • United States v. Armstrong: drug offenses
  • In re Winship: juvenile equivalent of larceny (theft)
  • Owens v. Maryland: driving while intoxicated
  • Bordenkircher v. Hayes: check forgery
  • Yates v. United States: concealing a tangible object related to government investigation

The cases listed above were selected to illustrate the process of criminalization, enforcement, and adjudication; they were not selected on the basis of the types of crimes that were charged. All the same, these cases have offered a useful sampling of activities that are frequently classified as criminal: the possession or distribution of drugs; taking property; risky activities (drunk driving); regulatory violations (failure to register); and, in just one of the above cases, physical violence. Most states organize their criminal codes into categories defined by the type of prohibited conduct: Offenses Against Property; Offenses Against the Person; Offenses Against Public Administration; Offenses Against Public Order; Offenses Against Public Health, Safety, and Morals; and so on. States do not always use the same labels or classify particular offenses under the same headings, but you will see some common patterns if you browse the tables of contents of a few state criminal codes, which is easy to do:

Most states post their codes online, so you can probably find yours easily even if it is not listed above. Browsing penal codes can help you gain a sense of the usual structure and content of codes, but it should also reinforce this point: you should not think of learning criminal law as simply a matter of learning the definitions of offenses. If someone wants to know the exact definition of burglary in Texas (to take one example), they can find it online relatively easily. As a lawyer, the value you bring to your clients will not come from your ability to recite the elements of any given statute, or even from your ability to describe typical patterns of burglary definitions—though it is indeed useful to be familiar with those patterns. Rather, your services as a lawyer will be valuable if you understand how the criminal system works and how to help your clients navigate through it. You will need to understand the key decisions to be made and the texts that will matter to those decisions, and you will need to be able to make persuasive arguments to the relevant decisionmakers. With that in mind, this chapter has two broad goals: it seeks to teach the basic components and usual definitions of property crimes, and it aims to use this category of offenses to expand and reinforce your broader understanding of how criminal law works.

Property crimes are, roughly, crimes that involve some sort of misappropriation or misuse of property. That is only a rough description, we should emphasize. Many offense definitions include both a misappropriation of property and some other core element, such as the infliction or threat of physical harm to a person. Robbery (usually defined as theft by use or threat of force) is a clear example, and robbery is often classified as a violent crime or a “crime against the person” rather than a property crime. Arson is also sometimes classified as a violent crime, or placed in a separate category of “crimes against the habitation.” For our purposes, though, “property crimes” is a useful label for an array of offenses that involve misappropriation or misuse of property. Robbery and arson are addressed in this chapter, rather than the next chapter on crimes against the person, because they both are based in part on concerns about property. But both robbery and arson could be – and often are – classified as “violent crimes” or crimes against the person. The next chapter explores in more detail the classification of crimes as violent.

Property crime is the focus here; what is property? That question is more difficult than it may first appear, and you’re likely to tackle it in a separate course focused on property law. For now, recognize that property includes not just land, money, and objects but also, sometimes, information or other intangibles. Happily, most of the cases we consider in this chapter concern relatively easily recognizable forms of property, such as a car, a purse, or money. Recognize also that ideas about property and ownership change over time. American law once treated certain persons – enslaved persons – as themselves a form of property, and criminal law was used to enforce the property rights of slaveholders. Today, to use someone else’s labor and then refuse to pay for it could itself be the crime of “wage theft.” It should also be noted that the very existence of the United States as an independent, sovereign nation is premised on the claim that European settlers eventually became the legitimate owners of the land they occupied, notwithstanding the fact that indigenous peoples had previously lived on and used that land. This chapter will not delve deeply into these important issues or radical critiques of property itself (such as Proudhon’s quip that “property is theft!”). Instead, for the most part this chapter will take for granted the determinations about what is property, and who is an owner, that have been made outside of criminal law. We will focus on the ways in which criminal law is used to enforce those determinations. But it is worth remembering that these determinations about property and owners are human judgments rather than natural truths. The idea that it is wrong to take property from an owner may seem natural and intuitive, but what constitutes property and who qualifies as an owner are political and legal questions.

Similarly, it is worth noting that there are deep moral and political questions about the distribution of property, and again criminal law operates mainly to enforce whatever answers to those questions have been reached in other arenas. Some fields of law or policy, such as tax law or social welfare spending programs, may openly embrace redistributive aims. The criminal law of property, in contrast, mostly aims to preserve existing distributions of property. Put differently, redistribution through self-help is disfavored by criminal law, as you’ll see in this chapter.

As we will see, the criminalization of property offenses may be motivated by any of several different goals. A legislature may wish to protect owners’ rights in possession and control of the things they own. But property crimes also often seem designed to protect somewhat more abstract interests in trust, security, or “civil order.” Again, the interests that criminal law has seemingly sought to protect have evolved over time. In cases in this chapter, you will find both appeals to the past and efforts to break from it. In particular, you’ll see the continuing influence of common law concepts even in a world of statutes, as courts often trace the development and evolution of property crimes in order to make sense of a modern-day statute.

Property crimes are a significant source of criminal convictions (about one-quarter of all felony convictions) and of prison sentences (about one-fifth of all prison sentences). In state prisons, where most incarcerated persons are held, property crimes are the second-most frequent source of a prison sentence, after offenses classified as “violent.” Drug crimes, discussed in more detail in Chapter Seven, are the third most frequent type of conviction among persons held in state prison. (In the federal system, property crimes are not quite as important as a source of incarceration; violent offenses, drug offenses, and “public order” offenses all generate more federal prison sentences.) For more granular details on property crimes as a source of imprisonment in comparison to other types of crime, you can consult the Prison Policy Initiative’s “Whole Pie” chart, referenced earlier in this book and available at https://www.prisonpolicy.org/reports/pie2020.html.

There’s one other reason that property crimes are important: for better or worse, they’re a particular favorite of the people who write multiple choice questions for the Multistate Bar Exam (MBE). The MBE often features several questions about larceny, embezzlement, burglary, and other property offenses, usually assuming common law definitions of those offenses rather than providing a specific statutory definition. You may wish to wait until you’re actively preparing for a bar exam to memorize the MBE’s definitions of property offenses. But learning the basic contours of various property offenses now, including traditional common law definitions of those offenses, will certainly make bar preparation easier when that time comes.

By the end of this chapter, you should be able to analyze and apply statutory or common law definitions of a number of property crimes: larceny, embezzlement, “theft” more generally, burglary, trespass, robbery, and arson. You should be familiar with interpretive questions that arise frequently in relation to these offenses. And, as always, you should see how criminalization, enforcement, and adjudication decisions interact with one another. Look for ways in which changes to the criminalization of property offenses have shaped the enforcement and adjudication of these offenses.

Common Law to Consolidation: Larceny and Beyond

California Penal Code § 211. Robbery

Robbery is the felonious taking of personal property in the possession of another, from his person or immediate presence, and against his will, accomplished by means of force or fear.

California Penal Code § 484. Theft defined

a) Every person who shall feloniously steal, take, carry, lead, or drive away the personal property of another, or who shall fraudulently appropriate property which has been entrusted to him or her, or who shall knowingly and designedly, by any false or fraudulent representation or pretense, defraud any other person of money, labor or real or personal property, or who causes or procures others to report falsely of his or her wealth or mercantile character and by thus imposing upon any person, obtains credit and thereby fraudulently gets or obtains possession of money, or property or obtains the labor or service of another, is guilty of theft. …

California Penal Code § 487. Grand theft defined

Grand theft is theft committed in any of the following cases:

a) When the money, labor, or real or personal property taken is of a value exceeding nine hundred fifty dollars ($950)…


The PEOPLE

v.

Demetrius Lamont WILLIAMS

Supreme Court of California
305 P.3d 1241

Aug. 26, 2013

KENNARD, J.

… On July 4, 2009, defendant Demetrius Lamont Williams entered a Walmart department store in Palmdale. Using either a MasterCard or a Visa payment card, which was re-encoded with a third party’s credit card information, defendant bought a $200 Walmart gift card from a recently hired cashier, who was filling in for a cashier on a break. Defendant then tried to buy three more gift cards from the same cashier. At that point, the regular cashier came back and, after learning of the previous transaction, told defendant of Walmart’s policy prohibiting the use of credit cards for purchases of gift cards. Defendant was permitted to keep the $200 gift card he had initially bought.

Defendant then went to a different cash register and again presented a re-encoded payment card to buy another $200 gift card. The transaction was observed by a Walmart security guard who, accompanied by another guard, asked defendant for the receipt and payment card used. Defendant complied. When told that the payment card’s last four digits did not match those on the receipt, defendant produced two other re-encoded payment cards, but their numbers did not match those on the receipt either.

Defendant began walking toward the exit, followed by the two security guards. When defendant was told to stop, he produced yet another re-encoded payment card, but this card’s last four digits also did not match those on the receipt. As defendant continued walking toward the exit, he pushed one of the guards, dropped some receipts, and started running away. After a brief struggle inside the store, the guards wrestled defendant to the ground and handcuffed him. Recovered from defendant’s possession were four payment cards issued by MasterCard and Visa. Also retrieved from defendant were several gift cards from Walmart and elsewhere.

Defendant was charged with four counts of second degree robbery (§ 211), one count of second degree burglary (§ 459), one count of fraudulent use of an access card (§ 484g), one count of grand theft (§ 487, subd. (a)), and three counts of forgery (§ 484i, subd. (b)), a total of 10 counts… Regarding the grand theft count, the court instructed the jury on grand theft by false pretenses. The jury found defendant guilty as charged, and the trial court sentenced him to a total prison term of 23 years eight months. The Court of Appeal reversed defendant’s forgery convictions for insufficient evidence and [stayed] imposition of the burglary sentence… [but affirmed] defendant’s robbery convictions.

As he did in the Court of Appeal, defendant here argues his robbery convictions should be reversed because robbery requires theft by larceny, whereas the theft he committed was by false pretenses. We agree.

II

Robbery is “the felonious taking of personal property in the possession of another, from his person or immediate presence, and against his will, accomplished by means of force or fear.” The term “felonious taking” originated in the common law and was later adopted in California’s robbery statute. At issue here is the meaning of “felonious taking.” Can that element of robbery be satisfied only by the crime of theft by larceny, as defendant argues? Or can it also be committed through theft by false pretenses, as the Attorney General contends?

To help us ascertain the meaning that the Legislature intended when it used the words “felonious taking” in California’s robbery statute, we need to examine that statute’s common law roots.

A. Crime of Larceny

California statutorily defines the crime of theft by larceny as the felonious stealing, taking, carrying, leading, or driving away of the personal property of another. That statutory definition reflects its English common law roots.

Unlike statutory law, whose authority rests upon an express declaration by a legislative body, the common law “consists of those principles and forms which grow out of the customs and habits of a people,” enshrined in law by virtue of judicial decisions. Much of the law developed in English courts was later applied in England’s American colonies and then, after independence, in this nation’s states. As used in this opinion, the term “common law” denotes a “body of judge-made law … developed originally in England.” And, as used here, the term “common law crime” means a “crime that [was] punishable under the common law, rather than by force of statute.”

The common law defined larceny as the taking and carrying away of someone else’s personal property, by trespass, with the intent to permanently deprive the owner of possession. Larceny was considered to be an offense less serious than robbery because of robbery’s additional requirement of personal violence against, or intimidation of, the victim. Not that the distinction made any difference to the accused: Under the common law, robbery and larceny were felonies, and all felonies were punishable by death.

… By [the late 18th century], English society and its judiciary had become troubled by that excessively harsh punishment for theft crimes. This concern led the English courts to limit the scope of larceny. For instance, it was held not to be larceny—and not a crime at all—if someone in lawful possession of another’s property misappropriated it for personal use (the later offense of embezzlement), or if someone acquired title to another’s property by fraud (the later offense of false pretenses). These limitations to the law of larceny made sense in light of that crime’s original purpose of preventing breaches of the peace; because embezzlement and false pretenses lacked larceny’s requirement of a “trespass in the taking,” they were viewed as less likely to result in violence.

Although common law larceny was in some ways narrowed to limit punishment by death, the scope of larceny was in other ways broadened to provide greater protection of private property. For instance, in 1799 an English court decision introduced the concept of “larceny by trick.” Larceny by trick … involves taking possession of another’s property by fraud.

[Again,] larceny requires a trespassory taking, which is a taking without the property owner’s consent. Although a trespassory taking is not immediately evident when larceny occurs “by trick” because of the crime’s fraudulent nature, English courts held that a property owner who is fraudulently induced to transfer possession of the property to another does not do so with free and genuine consent, so “the one who thus fraudulently obtains possession commits a trespass….”

The reasoning supporting larceny by trick’s inclusion within the crime of larceny—that fraud vitiates the property owner’s consent to the taking—was not extended, however, to cases involving the fraudulent transfer of title. Under the common law, if title was transferred, there was no trespass and hence no larceny. The theory was that once title to property was voluntarily transferred by its owner to another, the recipient owned the property and therefore could not be said to be trespassing upon it. … These subtle limitations on the common law crime of larceny spurred the British Parliament in the 18th century to create the separate statutory offenses of theft by false pretenses and embezzlement….

B. Crimes of Theft by False Pretenses and Embezzlement

… Britain’s 18th century division of theft into the three separate crimes of larceny, false pretenses, and embezzlement made its way into the early criminal laws of the American states. That import has been widely criticized in this nation’s legal community because of the seemingly arbitrary distinctions between the three offenses and the burden these distinctions have posed for prosecutors….

For instance, it was difficult at times to determine whether a defendant had acquired title to the property, or merely possession, a distinction separating theft by false pretenses from larceny by trick. It was similarly difficult at times to determine whether a defendant, clearly guilty of some theft offense, had committed embezzlement or larceny, as an 1867 Massachusetts case illustrates. There, a defendant was first indicted for larceny and acquitted; later, on the same facts, he was indicted for embezzlement and convicted; and thereafter, on appeal, his conviction was set aside on the ground that his offense was larceny, not embezzlement. Com. v. O’Malley, 97 Mass. 584 (1867).

In the early 20th century, many state legislatures, recognizing the burdens imposed on prosecutors by the separation of the three crimes of larceny, false pretenses, and embezzlement, consolidated those offenses into a single crime, usually called “theft.” The California Legislature did so in 1927, by statutory amendment. In a 1954 decision, this court explained: “The purpose of the consolidation was to remove the technicalities that existed in the pleading and proof of these crimes at common law. Indictments and informations charging the crime of ‘theft’ can now simply allege an ‘unlawful taking.’ [Citations.] Juries need no longer be concerned with the technical differences between the several types of theft, and can return a general verdict of guilty if they find that an ‘unlawful taking’ has been proved [Citations.]. The elements of the several types of theft included within section 484 have not been changed, however, and a judgment of conviction of theft, based on a general verdict of guilty, can be sustained only if the evidence discloses the elements of one of the consolidated offenses.” People v. Ashley (1954).

As we pointed out in Ashley, the California Legislature’s consolidation of larceny, false pretenses, and embezzlement into the single crime of theft did not change the elements of those offenses….

C. Elements of Robbery, Larceny, and Theft by False Pretenses and Their Application Here

We now consider the issue here: whether robbery’s element of “felonious taking” can be satisfied through theft by false pretenses, the type of theft defendant committed.

Robbery is “the felonious taking of personal property in the possession of another, from his person or immediate presence, and against his will, accomplished by means of force or fear.” Reflected in that statutory definition are larceny’s elements of “the taking of another’s property, with the intent to steal and carry it away.” The taking required in larceny, as in robbery, must be “felonious.”

By adopting in the robbery statute the phrase “felonious taking” that was used in the common law with regard to both robbery and larceny, the California Legislature in all likelihood intended to attach to the statutory phrase the same meaning the phrase had under the common law.

…[A]ll larceny at common law was a felony, and thus the common law defined larceny as a “felonious taking.” Because California’s robbery statute uses the common law’s phrase “felonious taking,” and because at common law “felonious taking” was synonymous with larceny, we conclude that larceny is a necessary element of robbery….

Two differences in the crimes of larceny and theft by false pretenses tend to support our conclusion that only theft by larceny, not by false pretenses, can fulfill the “felonious taking” requirement of robbery.

First, larceny requires “asportation,” which is a carrying away of stolen property. This element of larceny, although satisfied by only the slightest movement, continues until the perpetrator reaches a place of temporary safety. Asportation is what makes larceny a continuing offense. Because larceny is a continuing offense, a defendant who uses force or fear in an attempt to escape with property taken by larceny has committed robbery. Similarly, the Attorney General asserts that defendant committed robbery because he shoved the Walmart security guards during his attempt to flee the store after acquiring the store gift cards through theft by false pretenses.

But theft by false pretenses, unlike larceny, has no requirement of asportation. The offense requires only that “(1) the defendant made a false pretense or representation to the owner of property; (2) with the intent to defraud the owner of that property; and (3) the owner transferred the property to the defendant in reliance on the representation.” People v. Wooten (1996). The crime of theft by false pretenses ends at the moment title to the property is acquired, and thus cannot become robbery by the defendant’s later use of force or fear. Here, when defendant shoved the store security guards, he was no longer engaged in the commission of theft because he had already acquired title to the Walmart gift cards; therefore, defendant did not commit robbery.

… We now consider another significant difference between larceny and theft by false pretenses. …[L]arceny requires a “trespassory taking,” which is a taking without the property owner’s consent. This element of larceny, like all its other elements, is incorporated into California’s robbery statute. By contrast, theft by false pretenses involves the consensual transfer of possession as well as title of property; therefore, it cannot be committed by trespass. This is illustrated by the facts in a recent Court of Appeal decision, People v. Beaver (2010). There, the defendant staged an accident at his place of employment, a ski resort, to obtain medical expenses for a preexisting [knee injury]. The defendant was convicted of grand theft. The Court of Appeal reversed the conviction, holding that the jury was instructed on the incorrect type of theft—theft by larceny—and instead should have been instructed on theft by false pretenses. Beaver said: “The present matter did not involve a taking of property from another without his consent. [The ski resort] willingly paid for defendant’s medical treatment on the false representation that [it] had caused defendant’s injuries. This was theft by false pretenses, not larceny.” The essence of Beaver’s holding is this: Because the ski resort consented to paying for the defendant’s medical treatment, the defendant did not commit a trespassory taking, and hence did not commit larceny.

Here too defendant did not commit larceny. Walmart, through its store employees, consented to transferring title to the gift cards to defendant. Defendant acquired ownership of the gift cards through his false representation, on which Walmart relied, that he was using valid payment cards to purchase the gift cards. Only after discovering the fraud did the store seek to reclaim possession. Because a “felonious taking,” as required [for robbery] must be without the consent of the property owner, or “against his will,” and Walmart consented to the sale of the gift cards, defendant did not commit a trespassory (nonconsensual) taking, and hence did not commit robbery….

The dissent proposes a theory, not discussed in the parties’ briefs, to bring defendant within the robbery statute…. The gist of the dissent’s reasoning is this: Section 490a [of the California Penal Code] says any law or statute that refers to or mentions larceny or stealing must be construed as meaning “theft”; although the robbery statute does not expressly mention larceny or stealing, it refers to them indirectly through the words “felonious taking,” which should be interpreted under § 490a as meaning “theft,” a crime that includes theft by false pretenses. Therefore, the dissent concludes, the “felonious taking” element in the robbery statute encompasses defendant’s conduct in this case.

The dissent’s theory would require us to conclude that, by enacting § 490a, the Legislature intended to alter two of the substantive elements of robbery: asportation and a trespassory taking. But the 1927 legislation enacting § 490a and the theft consolidation statute (§ 484) left unchanged the elements of theft. We are not persuaded that the Legislature intended to alter the elements of robbery, to which § 490a makes no reference whatever, while also intending to leave intact the elements of theft, to which it explicitly refers. As this court said more than 80 years ago, “the essence of § 490a is simply to effect a change in nomenclature without disturbing the substance of any law.People v. Myers (1929).

III

In resolving many complex legal issues, as Justice Oliver Wendell Holmes, Jr., observed, “a page of history is worth a volume of logic.” To determine the meaning of the words “felonious taking” in our statutory definition of robbery, we have delved into the sources of this statutory definition and, in turn, into the history of the common law crime of larceny and the statutory crime of theft by false pretenses. This review has led us to conclude that the words “felonious taking” in the robbery definition were intended to refer only to theft committed by larceny and not to theft by false pretenses.

The logic and fairness of this conclusion may be open to question because a thief who uses force to resist capture may be equally culpable whether the theft was committed by larceny (for example, ordinary shoplifting) or by false pretenses (as occurred here). Nevertheless, our task is simply to interpret and apply the laws as the Legislature has enacted them, not to revise or reform them to better reflect contemporary standards.

We reverse the Court of Appeal’s judgment upholding defendant’s four robbery convictions. Because other aspects of the Court of Appeal’s decision may be affected by our reversal of defendant’s robbery convictions, the matter is remanded to that court for further proceedings consistent with the views expressed in this opinion.

Dissenting opinion by BAXTER, J.

…[The majority’s] reasoning and result contradict[] the legislative intent behind California’s robbery and unified theft statutes. is in conflict with long-standing California jurisprudence, including several decisions of this court that have reached the opposite conclusion. And it is patently at odds with the important public policies served by the robbery statute. “Robbery violates the social interest in the safety and security of the person [robbed] as well as the social interest in the protection of property rights.” Both interests are implicated when a thief enters a business establishment, steals property, and then uses force or fear against a robbery victim or victims while fleeing, regardless of the particular manner of theft employed. I respectfully dissent.

… At the same time as the 1927 consolidation of all common law forms of theft into a unified “theft” crime (§ 484), our Legislature also enacted this provision: “Wherever any law or statute of this state refers to or mentions larceny, embezzlement, or stealing, said law or statute shall … be read and interpreted as if the word ‘theft’ were substituted therefor.” (§ 490a).

Section 490a indicates the Legislature’s intent that the different types of common law theft consolidated in § 484 are to be treated as the single crime of “theft” in California…

True, unlike the burglary statute, the robbery statute does not utilize either the term “larceny,” or the term “stealing.” Rather, it uses the broader phrase “felonious taking of personal property” to denote the taking element of robbery. Section 490a, however, states that any law or statute that “refers to or mentions larceny … or stealing ” (italics added) should be read and interpreted as if the word “theft” were substituted. Thus, the statute need not specifically mention larceny or stealing; to simply refer to larceny or stealing is enough. A felonious taking is a taking done with the intent to steal another’s property…

In short, the robbery statute is a statute that “refers to … larceny or stealing.” (§ 490a, italics added.) That is because the “felonious taking” element of robbery is a taking done with the intent to steal another’s property “against his will.” Because § 490a directs that any law that “refers to … larceny or stealing” is to be read and interpreted as if the term “theft” was inserted therein, and because the robbery statute incorporates such a reference, albeit indirectly, the “felonious taking” element of robbery must be interpreted as synonymous with “theft.” …Here, defendant’s conduct in stealing gift cards from Walmart, although accomplished by false pretenses, plainly satisfied the felonious taking element of robbery.

The majority’s analysis of the 18th century English common law roots of the various common law forms of theft … in support of its conclusion that the common law crime of theft by false pretenses is not a continuing form of theft, and cannot be transformed into robbery where force or fear is later used, overlooks the important remedial legislation that consolidated the common law forms of theft into the unified crime of “theft” in California….

 I would affirm the judgment of the Court of Appeal.


Notes and questions on People v. Williams

  1. Williams offers both an overview of common law theft offenses and an alternative statutory approach that is typical of many contemporary criminal codes. Note the multiple different offenses mentioned in the majority’s discussion of the common law: larceny, robbery, theft by false pretenses, embezzlement, and larceny by trick. It may be helpful to identify the different elements of these offenses as they were typically defined at common law. To do so, it is useful to keep in mind a distinction between possession of property (having immediate control over it) and holding legal title to it (being recognized as the owner by law). Possession and ownership can coincide, but they can also diverge. If you’ve allowed me to borrow your car, I’m in temporary possession of it but I’m not the owner. The chart below captures the common law definitions of property offenses as identified by the California court in Williams. Each of these offenses will be discussed in more detail later in this chapter, but comparing the definitions can help you get used to thinking of crimes in terms of their elements.

    Larceny

    • Taking (by trespass)
    • Carrying away
    • The property
    • Of another
    • With intent to permanently deprive the owner of possession

    Robbery

    • Larceny
    • By force

    False pretenses

    • Acquiring title
    • To the property
    • Of another
    • By fraud

    Embezzlement

    • While in possession
    • Of the property
    • of another,
    • Converting that property to personal use
    • By fraud

    Larceny by trick

    • Taking (by fraud, as a form of trespass)
    • And carrying away
    • The property
    • Of another
    • With intent to permanently deprive the owner of possession

     

  2. According to the Williams court, larceny and related offenses emerged not primarily out of a concern to protect property rights, but rather with the “original purpose of preventing breaches of the peace.” It was taking property in a way likely to produce violence or conflict that was criminalized; appropriation of property by fraud or secrecy was not initially seen to warrant criminal intervention. Today, criminal law is concerned with both force and fraud, and with the protection of property rights even when neither force nor fraud is deployed. Think about how and why societies have made different criminalization choices over the centuries. Both in the ancient past and here in the twenty-first century, people have disagreed about whether violence is categorically worse than deception. For example, after Bernie Madoff was convicted of fraud offenses involving over 64 billion dollars, one of the investors defrauded by Madoff invoked the ancient Italian poet Dante Aligheri, whose Divine Comedy famously imagines the descending circles of hell. According to Dante, those who commit fraud are subject to even more severe divine punishments than those who use violence. Though violence is certainly terrible, fraud was still more displeasing to God, since “the vice of fraud is man’s alone.” See Dante, The Divine Comedy; see also Burt Ross, What I Told Madoff Today, Daily Beast (June 29, 2009).
  3. Common law larceny required both i) a taking (sometimes called “caption”) by trespass, otherwise known as a taking without consent, and ii) “asportation,” or the carrying away of the property. In most states, these somewhat archaic concepts have been replaced with the unified concept of “possession or control.” For more details, see State v. Donaldson, the next case in this chapter.
  4. At the beginning of Part III of its opinion, the Williams majority quotes Oliver Wendell Holmes, Jr.: “[A] page of history is worth a volume of logic.” And the court then concedes that “the logic and fairness of [our] conclusion may be open to question….” Is the court sacrificing fairness unnecessarily in order to preserve dated and obscure legal concepts? Or is the reversal of Williams’s robbery convictions a fair outcome after all? Is the court’s decision dictated by earlier criminalization decisions, and if so, which ones—common law decisions or legislative decisions?  As you read the cases in this chapter, consider whether logic and history are in tension in theft law, and if so, which has prevailed.

Check Your Understanding (5-1)

For the Williams majority (but not the dissent), common law categories and concepts continue to be relevant even for a modern theft statute. For a different assessment of the relevance of the common law, consider State v. Donaldson, below. It may be helpful first to consider the text of the Iowa statute applied in Donaldson, along with the text of the Model Penal Code provision on which the Iowa statute is based.

Iowa Code § 714.1. Theft defined

A person commits theft when the person does any of the following:
1. Takes possession or control of the property of another, or property in the possession of another, with the intent to deprive the other thereof.

Model Penal Code § 223.2. Theft by unlawful taking or disposition

(1) Movable Property. A person is guilty of theft if he unlawfully takes, or exercises unlawful control over, movable property of another with purpose to deprive him thereof.

(2) Immovable Property. A person is guilty of theft if he unlawfully transfers immovable property of another or any interest therein with purpose to benefit himself or another not entitled thereto.


STATE of Iowa, Appellee

v.

Dean Lester DONALDSON, Appellant

Supreme Court of Iowa
663 N.W.2d 882

June 11, 2003

STREIT, Justice.

…At 1:50 a.m., a Sioux City police officer saw a van parked in front of Combined Pool & Spa with its sliding door partially open. The officer illuminated the van. As he walked towards the van, the brake lights flashed. Two men hotfooted across Highway 75. The officer gave chase, but was unable to find them. Upon returning to his squad car, the officer saw the steering column in the van had been forcibly removed and there were wires protruding from it. The radio was on and the “check engine” sign was lit on the console. Later, one of the men was found and identified as Dean Lester Donaldson.

Donaldson was charged with one count of second-degree theft as an habitual offender. Prior to the trial, Donaldson filed a motion to adjudicate law points arguing the facts did not support a charge of theft. Donaldson asserted because he never possessed the van, he could not be convicted of theft. Donaldson argued, at most, the facts supported a charge of attempted theft. However, Iowa does not recognize a separate crime of attempted theft. The State asserted Donaldson took possession of the van when he hot-wired it. The district court agreed with the State and denied Donaldson’s motion… After a trial, Donaldson was convicted of second-degree theft. Prior to sentencing Donaldson renewed his motion raising the same arguments in the original motion to adjudicate law points. The district court overruled the motion and sentenced Donaldson. Donaldson appeals.

…This appeal is limited to one main issue. We must determine whether the district court properly denied Donaldson’s motion for judgment of acquittal challenging the sufficiency of the facts to support a conviction of second-degree theft. The question is whether Donaldson possessed or controlled another’s van when he broke into it, dismantled the steering column, and manipulated the ignition switch turning the radio on, lighting the “check engine” sign, and causing the brake lights to flash. Our review is for correction of errors of law.

…The State charged Donaldson with second-degree theft pursuant to Iowa Code § 714.1(1) [and with other offenses]. This statute provides “a person commits theft when” he or she “[t]akes possession or control of the property of another, or property in the possession of another, with the intent to deprive the other thereof.” At the end of the State’s case, Donaldson moved for a judgment of acquittal. He argued the State failed to prove the elements of theft…. Counsel argued the “starter must be engaged for there to be actual control over that vehicle.” The court disagreed and overruled Donaldson’s motion.

The Iowa theft statute is modeled after the Model Penal Code, with slight variation. Model Penal Code § 223.2. Our terms “possession or control” of another’s property replace the common law larceny requirements of “caption” and “asportation.” “Caption,” or taking, occurred when the actor secured dominion over the property of another. The element of “asportation,” or carrying away, was satisfied with even the most slight change in position of the stolen object. At common law, to prove a theft, the State had to show a defendant took the property of another, i.e., secured dominion over it, and carried the property away.

The asportation requirement was important at common law because if a defendant’s actions fell short of causing the object of the theft to move, the defendant was guilty of attempt only. Because a completed larceny was generally a felony whereas attempt was a misdemeanor, significant differences in “procedure and punishment turned on the criminologically insignificant fact of slight movement of the object of theft.” In modern criminal law, however, the penal consequences between attempt and a completed theft are so minimal that it has become less important to draw a bright line between the two actions. As such, the element of asportation is no longer necessary.

Iowa, like many other states following the Model Penal Code, has abandoned the common law asportation requirement. The key to our statute is the words “possession or control.” In determining the meaning of “possession” and “control,” we look to the Model Penal Code for guidance as our statute is modeled after it. The Model Penal Code contemplates “control” of the object to begin when the defendant “use[s] it in a manner beyond his authority.” The method of exerting control over the object of the theft is important only insofar as it “sheds light on the authority of the actor to behave as he did.” Our statute replaces the common law element of “taking” with “possession.” The Model Penal Code provides a person commits theft if he or she “unlawfully takes, or exercises unlawful control over” the property of another. A taking in this sense concerns whether the offender exerted control over the object “adverse to or usurpatory of the owner’s dominion.” That is, one possesses an object if he or she secures dominion over it. To summarize the above concepts, “possession or control” begins and a theft is completed when the actor secures dominion over the object or uses it in a manner beyond his authority.

Donaldson argues his conduct, at most, is sufficient to prove attempted theft, not a completed theft. We acknowledge the issue before us is complicated because “all theft partakes of the character of attempt.” The line between attempt and a completed theft is a thin one. “The thief proposes to make the property his own more or less permanently; but he is nonetheless a thief if, shortly after he exerts his dominion over the property of another, he is prevented from making off with it.”…

The question before us concerns whether the defendant possessed or controlled the object of the theft. The critical issue, as the statute dictates, is not whether the defendant used or operated the object of the theft. As to Donaldson’s conduct, we must determine whether he exercised wrongful dominion or unauthorized control of the van. The judge instructed the jury on “possession” using the Iowa Criminal Jury Instructions.[1] Bearing in mind the definitions of “control” and “possession” as contemplated by the Model Penal Code, we turn to the facts.

The undisputed facts of the case [establish that] Donaldson entered a van owned by Combined Pool & Spa…. As the officer approached, Donaldson got out of the driver’s side and ran away. The officer called after Donaldson, identified himself as a police officer, and ordered him to stop. Donaldson kept running. When the officer checked the van, he saw the steering column had been forcibly dismantled; there were wires hanging from the column. The ignition switch had been removed. The radio was operating. The “check engine” sign on the dashboard was lit. At trial, one of the officers testified Donaldson had engaged all of the electric systems. After turning on the electric accessory systems in the car, according to the officer, all Donaldson had left to do was engage the starter.

There is no evidence in the record to suggest Donaldson’s tearing apart the steering column was intended for any purpose other than to deprive the owner of her possession of the van. Donaldson argues he did not possess or control the van because he did not have the “ability to readily move or remove” it. This, however, is not the test for possession or control. Because we have abandoned the common law asportation requirement, movement or motion of the car is not essential to finding a defendant had possession or control of the car. Our theft statute does not state possession or control is tantamount to “operation” of the object of the theft. To interpret our statute in this manner is to restrict the definition of theft more narrowly than the legislature intended. Given a strict interpretation of the statute, the State only had to show Donaldson had control of the van, i.e., he had dominion over it in a manner inconsistent with his authority. We are unwilling to imply an “operation” requirement for certain kinds of property that are normally operated by its possessor.

The mere fact that Donaldson was interrupted by the police officer before he engaged the starter motor does not remove this case from the realm of a completed theft. It is not necessary that the engine was running and the van could have been moved. That is, technical operation of the van is not necessary to find Donaldson exercised wrongful dominion or unauthorized control over the van. … Certainly, Donaldson’s acts were sufficient to set into motion the steps necessary to power the van. It was not necessary that the engine was actually running. Rather, at the moment Donaldson began to manipulate the electrical wires for the purpose of starting the engine, he exerted complete control over the vehicle.

In sum, the facts before us show Donaldson was using the van owned by another person. He had the power and intention at the given time to exercise unfettered dominion over the van. Donaldson was in a position to exclude all others from the van, for example, by locking it. No one else could have hot-wired the van or started it with a key while Donaldson had control over it. Moreover, he used the van without the owner’s consent and in a manner beyond his authority. Donaldson entered the company’s van around 1:30 in the morning. He tore apart the steering column. The ignition switch had been removed; wires protruded from the ignition. The brake lights flashed. The radio worked. The “check engine” sign was lit. When the officer approached the van, Donaldson got out of the driver’s side and ran away. All of these facts together are sufficient to show Donaldson controlled the van within the meaning of § 714.1(1). As such, the trial court properly denied Donaldson’s motion for judgment of acquittal. We affirm.

[The court noted in a footnote that in future theft prosecutions under 714.1, “the district court should sculpt its jury instructions using the concepts articulated in the Model Penal Code. The jury should be instructed a theft is completed when the defendant secures dominion over the object of the theft or uses it in a manner beyond his authority.”]


Notes and questions on State v. Donaldson

  1. In Chapter Seven, which covers gun and drug offenses, we will study the concept of “possession” in more detail. This case gives you a preview. The Donaldson court says that “possession” for purposes of property offenses should be defined differently than “possession” as that term is used in contraband offenses. What are the key differences between the different definitions of possession? Are the differences significant enough that Donaldson’s conviction should have been reversed due to inadequate jury instructions? If not, why does the court advise the use of different instructions in future theft prosecutions?
  2. In the second half of the twentieth century, inspired in part by the Model Penal Code, many U.S. jurisdictions consolidated the various narrowly defined property offenses that had existed at common law and created one new, broader offense called “theft.” But in some states, like California, the old common law categories have continued to influence judicial interpretations of the new consolidated crime, as you saw in People v. Williams, above. In other states, including Iowa as illustrated by Donaldson, courts have viewed the consolidation of theft as a more substantial redefinition of property crimes. Had Dean Lester Donaldson “hot-wired” this van in California rather than Iowa, would he have been guilty of theft under Cal. Penal Code § 484?
  3. A related question: does the Model Penal Code approach change the scope of property offenses? That is, is the same range of conduct treated as criminal under the common law definitions and the MPC, or does the MPC broaden (or narrow) the scope of liability for property offenses?
  4. To help you assess the previous question, consider Lee v. State, 474 A.2d 537 (Ct. Sp. App. Md. 1984). Lee was a shoplifting case: the defendant put a bottle of liquor into his trousers in a store, was approached by an employee, and then put the bottle back on the shelf and fled the store. Was this conduct “theft”? The Lee court, like the courts in Williams and Donaldson, discussed in detail the history of common law property offenses, and the eventual consolidation of these offenses into one crime of theft. The court argued that consolidation created broader criminal liability for shoplifting than would have existed under common law.

    In Maryland… [s]everal separate offenses, each involving some sort of taking and carrying away of property with an intent to deprive the owner, were consolidated under Article § 27… The legislature consolidated these offenses in an effort to eliminate the “technical and absurd distinctions that have plagued the larceny related offenses and produced a plethora of special provisions in the criminal law.” …The evolution of theft law is particularly relevant to thefts occurring in modern self-service stores where customers are impliedly invited to examine, try on, and carry about the merchandise on display. In a self-service store, the owner has in a sense, consented to the customer’s possession of the goods for a limited purpose. Under common law principles of theft, a person could not have been convicted if apprehended while still in the store because the perpetrator would have rightful possession (albeit temporarily) and thus could not perform the element of trespassory taking until he left the store without paying (at which point it might be too late). Under the present law, the fact that the owner temporarily consents to possession does not preclude a conviction for larceny if the customer exercises dominion and control over the property by using or concealing it in an unauthorized manner. Such conduct would satisfy the element of trespassory taking as it could provide the basis for the inference of the intent to deprive the owner of the property.

    Lee, 474 A.2d at 540-541. 

  5. Both the Williams and Donaldson courts observe that one explicit aim of consolidation was easing the path of enforcement and making convictions easier to obtain. If indeed the consolidation of theft offenses resulted in an expansion of the scope of criminal law, then consolidation has not simply eased the burden on enforcement officials: it has increased their power and discretion. With that in mind, note that arrests, convictions, and sentences for property offenses show similar patterns of racial disparity as those identified for violent crimes and drug offenses. Racial disparities in the enforcement of property offenses have not (so far) received nearly as much scholarly or public attention as racial disparities in the enforcement of drug offenses, but there is evidence that enforcement officials do target persons of color for theft prosecutions more often than white persons who engage in similar conduct. For example, one study found that while persons who reported (anonymously) that they had engaged in shoplifting were overwhelmingly young and white, the individuals who were actually prosecuted for shoplifting were mostly “very old or very young Hispanic or black men.” Rachel Shteir, The Steal: A Cultural History of Shoplifting 88-89 (2011). Some commentators describe the increased surveillance and detention of minority shoppers as the de facto criminalization of “shopping while black.” Id.
  6. The Donaldson court observes, “‘All theft partakes of the character of attempt.’ The line between attempt and a completed theft is a thin one.” This observation raises difficult questions about the timing and completion of crimes. We will examine these questions in much greater detail in Chapter Eight, which addresses inchoate crimes including attempt. For now, notice that neither common law definitions of property crimes nor modern statutory definitions require the defendant to keep property permanently in order to be guilty of the offense. A defendant completed common law larceny by “taking” and “carrying away” the property (with the right mental state), but carrying away (also called asportation) could be achieved simply by a slight change in the position of the property, as the Donaldson court notes. For the Iowa statute applied in Donaldson, asportation is not necessary, and the crime of theft is complete as soon as the defendant “takes possession or control” of the property (with the right mental state).
  7. As the previous note explains, most definitions of property offenses do not require the defendant to keep the property permanently. However, many definitions do require an intention to keep the property permanently. See, for example, the definition of larceny in People v. Williams earlier in this chapter: “The common law defined larceny as the taking and carrying away of someone else’s personal property, by trespass, with the intent to permanently deprive the owner of possession” (emphasis added). Many courts characterize this intent requirement as a “specific intent” requirement.
  8. In People v. Perry, 864 N.E.2d 196 (Ill. 2007), a defendant was charged with theft by deception after he occupied a hotel room for over three months but did not pay the bill. Under the Illinois statute, “A person commits theft when he knowingly … obtains by deception control over property of the owner … and intends to deprive the owner permanently of the use or benefit of the property.” Is occupancy of a hotel room “property”? And did a defendant who did not intend to stay in the room forever act with the requisite intent to permanently deprive the owner? The Illinois Supreme Court upheld the defendant’s conviction, finding that “[t]he property at issue here is the use of a hotel room. … One night in one room is a thing of value. When this thing of value is taken by deception, the owner has permanently lost the benefit of one night’s income.” Id. at 211.
  9. Under the reasoning of Perry in the previous note, could non-payment of rent be a criminal offense? In fact, in almost every jurisdiction, rent disputes (including non-payment) are classified as civil matters. In Arkansas, however, a landlord can file a notice to vacate if a tenant is late with a rent payment, and if the tenant does not vacate within ten days, a warrant is issued for arrest. This “criminal evictions” law has drawn criticism from civil rights advocates for years, in part because the tenants prosecuted are disproportionately poor Black women. See Human Rights Watch, Pay the Rent or Face Arrest: Abusive Impacts of Arkansas’s Draconian Evictions Law (2013). As of 2022, a challenge to the Arkansas law is pending in federal court.
  10. Arkansas’s criminal evictions statute is one of a kind. But the imposition of criminal liability for a failure to pay fees or other assessments is quite common. Commentators have used the term “the criminalization of poverty” to describe the heavy fees often assessed of criminal defendants and the additional sanctions imposed when defendants do not pay. The same phrase is also sometimes used to critique cash bail systems, which often require defendants without money to choose between lengthy pretrial detention or a quick guilty plea. See also Kaaryn Gustafson, The Criminalization of Poverty, 99 J. Crim. L. & Criminology 643 (2009) (discussing “the social construction of welfare fraud” and ways in which the welfare system and the criminal legal system have become increasingly intertwined).

The relevant statute for the next case is included within the text of the court’s opinion.


 

STATE of Florida, Appellant

v.

David Paul SIEGEL, Appellee

District Court of Appeal of Florida, Fifth District
778 So.2d 426

Feb. 2, 2001

THOMPSON, C.J.

…[David Paul] Siegel was charged by Information with grand theft. This charge followed Siegel’s expulsion from the University of Central Florida (UCF) for submitting fraudulent financial vouchers as a member of student government. The Information charged that Siegel:

[D]id … knowingly obtain or use, or endeavor to obtain or use a computer and computer equipment, of a value of THREE HUNDRED DOLLARS ($300.00) or more, the property of another, to-wit: UNIVERSITY OF CENTRAL FLORIDA … as owner or custodian thereof, with the intent to temporarily or permanently deprive said owner or custodian of a right to the property or a benefit therefrom, or to appropriate the property to the defendant’s own use or to the use of a person not entitled thereto.

Both sides agree that Siegel was allowed to use, as part of his responsibilities as a member of the UCF student government, an IBM Thinkpad 755 CDV (laptop computer) owned by UCF. At some point, UCF officials demanded that Siegel return the laptop computer pursuant to UCF Student Government Laptop Policy. … Siegel refused to return the computer and, among other things, this criminal case resulted from that refus[al].

[Under Florida law,] a defendant may move for dismissal [by alleging] that “[t]here are no material disputed facts and the undisputed facts do not establish a prima facie case of guilt against the defendant.” Under this rule it is the defendant’s burden to specifically allege and swear to the undisputed facts in a motion to dismiss and to demonstrate that no prima facie case exists upon the facts set forth in detail in the motion. The purpose of this procedure is to avoid a trial when there are no material facts genuinely in issue.

The trial court dismissed this case, according to the record, because when Siegel first received the laptop computer, he did not have the criminal intent to deprive UCF of the computer. The theft statute Siegel is charged under provides in part:

A person commits theft if he or she knowingly obtains or uses, or endeavors to obtain or to use, the property of another with intent to, either temporarily or permanently: (a) Deprive the other person of a right to the property or a benefit from the property.

§ 812.014(1)(a). In defining “obtains or uses,” the theft chapter provides in pertinent part:

“Obtains or uses” means any manner of:

…[c]onduct previously known as stealing; larceny; purloining; abstracting; embezzlement; misapplication; misappropriation; conversion; or obtaining money or property by false pretenses, fraud, or deception….

§ 812.012(2)(d) 1., Fla.Stat. (emphasis added). In certain types of theft cases, like larceny or false pretenses, criminal intent must be formed at the time of the original taking… Under [this statute], however, theft is more than just larceny or theft by false pretenses. Theft also includes the common-law crime of embezzlement.

[In a footnote, the court explained:

Embezzlement “may be defined as: (1) the fraudulent (2) conversion of (3) the property (4) of another (5) by one who is already in lawful possession of it.” LaFave & Scott, Criminal Law at § 8.6. Under the state’s theory of the case, Siegel lawfully possessed UCF’s laptop computer, but refused to return it when the owner of the computer requested that he do so. This conduct arguably worked as an attempt to fraudulently convert the computer to Siegel’s possession.]

Unlike the crimes of larceny and false pretenses, embezzlement does not require that the defendant have criminal intent when he obtains the property in question. The alleged facts, if proven, fit the crime formerly known as embezzlement and now known as theft under the omnibus theft statute.


Notes and questions on State v. Siegel

  1. This case has a somewhat different procedural history than many of the appellate opinions included in this book, because it does not involve an appeal after a conviction. In this case, the trial court dismissed the charges against Siegel before any trial took place, and the prosecution then appealed that dismissal. Notice Siegel was charged with grand theft by an “Information,” which is quoted in the court’s opinion. Unlike an indictment, an information does not require the prosecutor to submit the charges to a grand jury for approval. But there may exist other limits on the prosecutor’s discretion. Here, Siegel argued that the facts as alleged in the Information did not constitute the offense of grand theft.
  2. Siegel was charged with grand theft, not a crime called embezzlement. Be sure you understand how the definition of embezzlement becomes important in this case. As you’ve seen earlier in this chapter, many states interpret their general “theft” statutes with reference to common law definitions of property crimes. The theft statute used to charge Siegel includes an element that the defendant “obtains or uses” the property of another. And a portion of the Florida statute provides that “obtains or uses” includes conduct previously known as embezzlement.
  3. Look carefully at the elements of embezzlement as described by the Florida court. To commit this crime, the defendant must already be in “lawful possession” of the property in question. For example, an employee who is entrusted with company funds may be in lawful possession of that money. But if he takes the money for personal use, he may be guilty of embezzlement.
  4. In other words, embezzlement does not require that the defendant have any wrongful intention at the time that he or she first takes possession of the property. That becomes important in this case. Siegel argued that he did not commit theft because he had no criminal intent when he first took possession of the laptop. For many theft crimes, this lack of criminal intent would be a plausible defense, thanks to a principle sometimes referred to as “concurrence of the elements.” This principle requires that the prosecution must establish all necessary elements of the offense and show that they occurred simultaneously: the defendant must have held the requisite mental state at the time that the conduct elements took place. If Siegel “obtained” the laptop only at the moment that he first took possession of it, then he did not have the right mens rea at the time of the actus reus. But if Siegel’s ongoing possession was an ongoing act of “obtaining or using” the property, then it does not matter if Siegel formed the intent to keep the laptop for his own personal use only later.

10 U.S.C. § 921. Uniform Code of Military Justice, Article 121. Larceny and wrongful appropriation

(a) Any person subject to this chapter who wrongfully takes, obtains, or withholds, by any means, from the possession of the owner or of any other person any money, personal property, or article of value of any kind–

(1) with intent permanently to deprive or defraud another person of the use and benefit of property or to appropriate it to his own use or the use of any person other than the owner, steals that property and is guilty of larceny; or

(2) with intent temporarily to deprive or defraud another person of the use and benefit of property or to appropriate it to his own use or the use of any person other than the owner, is guilty of wrongful appropriation.

(b) Any person found guilty of larceny or wrongful appropriation shall be punished as a court-martial may direct.


UNITED STATES

v.

Andrew S. COFFMAN, Lance Corporal (E–3), U.S. Marine Corps

U.S. Navy–Marine Corps Court of Criminal Appeals
62 M.J. 676

Decided 22 Feb. 2006

DORMAN, Chief Judge:

A military judge sitting as a special court-martial convicted the appellant, pursuant to his pleas, of a false official statement and larceny….

The appellant pleaded guilty to the theft of several items of special operations equipment, such as a force vest, canteen covers, and a duty belt (hereinafter referred to as “gear”). The total value of the stolen gear exceeded $500.00. The gear belonged to another Marine…

The appellant was serving in Al Hillah, Iraq, when he took the gear he was charged with stealing. The gear was located in an open box, and, at the time he took it, the appellant did not know who owned the gear. The appellant found the box in a room that he and others had been told to clean out in preparation for another platoon’s arrival. Unit personnel had previously used this room to store their packs. The room contained several boxes that they had been instructed to dispose of, including the box containing the gear. As they cleaned out the room, they discovered items that were never picked up by their owners and appeared to have been left behind for trash. The appellant took the box from a room where unit personnel had been storing their packs and he brought it to his rack. There was no name on the gear, but the appellant knew it did not belong to him. The appellant was the first one to find the box containing the gear. The appellant also knew that the items should not have been discarded. He went up and down the passageway asking whether anyone had left a box of gear in the room. He asked almost the entire platoon. When he could not determine who owned the gear, he decided to use it himself.

The appellant used the gear for about a month while going on patrols. The use continued until his section leader confronted him about whether the gear belonged to him. Initially, the appellant told the section leader that he had purchased the gear. This false statement was prosecuted under Article 107, UCMJ. The appellant did not learn who owned the gear until after he had surrendered it. The owner was a member of the appellant’s battalion, and the appellant was acquainted with him. The appellant informed the military judge that if he had not been confronted by the command, he would have continued to use the gear.

As the providence inquiry continued, the appellant then answered “yes” or “no” to a series of questions dealing with the legality of his actions. He admitted that he knew it was wrongful to take the gear, that the gear was not abandoned, that he intended to permanently deprive the owner of the gear, that he had no legal justification or excuse for his actions, and that he took and retained the gear with a criminal state of mind.

… The appellant now argues that his plea is improvident because the military judge failed to adequately inquire into the “apparent defense of ignorance or mistake of fact as to whether the gear … was abandoned, lost, or mislaid.”…

… Before accepting a guilty plea, the military judge must explain the elements of the offense and ensure that a factual basis for the plea exists. Mere conclusions of law recited by the accused are insufficient to provide a factual basis for a guilty plea. The accused “must be convinced of, and able to describe all the facts necessary to establish guilt.” Acceptance of a guilty plea requires the accused to substantiate the facts that objectively support his plea.

…The standard of review to determine whether a plea is provident is whether the record reveals a substantial basis in law and fact for questioning the plea. Such rejection must overcome the generally applied waiver of the factual issue of guilt inherent in voluntary pleas of guilty, and the only exception to the general rule of waiver arises when an error materially prejudicial to the substantial rights of the appellant occurs. … An abuse of discretion standard is applied in reviewing the question of whether a military judge erred in accepting a guilty plea. In considering the adequacy of guilty pleas, we consider the entire record…

In our review of the record, we determined that the military judge accurately listed the elements of larceny and defined the terms relevant to those elements. We also determined that the appellant indicated an understanding of the elements of the offense and that he acknowledged that they correctly described what he did. Thereafter, the military judge conducted an inquiry with the appellant to determine whether a factual basis for the plea existed. The inquiry went well until such time as the appellant essentially informed the military judge that the gear he took had been left in the room as trash. After that point, most of the questions asked by the military judge called for a “yes” or “no” answer, and many called for legal conclusions.

Abandoned property cannot be the subject of a larceny. The appellant’s statement to the military judge that the gear had been left there as trash raised the issue of mistake of fact. Furthermore, since larceny is a specific intent offense, if the appellant had an honest belief that the property was abandoned, he has a complete defense.

For a complex offense such as conspiracy, robbery, or murder, a failure to discuss and explain the elements of the offense during the providence inquiry has been held to be fatal to the guilty plea on appeal. Similarly, a military judge should explain the elements of defenses, such as mistake of fact and abandonment, if raised by the appellant during the providence inquiry. Failure to do so can leave unresolved substantial inconsistencies in the pleas and/or raise questions concerning whether the appellant was armed with sufficient information to knowingly plead guilty. Where the elements of an offense, or defenses, are commonly known by most servicemembers, however, it is not necessary for the military judge to explain them, if it is otherwise apparent on the record that the accused understood the elements or the defense.

In the case before us, the military judge failed to explain the mistake of fact defense to the appellant. Although the military judge did ask the appellant if he believed the gear was abandoned, he did not provide the appellant with the legal definition of abandoned property. A reading of the case law with respect to this issue makes clear that the legal significance of the term “abandoned” is not one that would be “commonly known and understood by servicemembers.”

Applying the standards of review noted above, we conclude that the record reveals a substantial basis in law and fact to question the appellant’s guilty plea to larceny. Thus, we conclude that the military judge erred by failing to inform the appellant of the defense of mistake of fact and the definitions and legal significance of abandoned property. He did not adequately resolve the issue of mistake of fact. When the appellant informed the military judge that the gear had been left behind as trash, the military judge inappropriately asked the appellant “yes” or “no” type questions that called for legal conclusions. By not explaining the relevant legal terms, the military judge denied the appellant the ability to make an informed decision concerning the answers he provided. In light of these errors, we conclude that the appellant’s guilty pleas to Charge II and its specification are not provident.

We take this opportunity to note that the error in this case does not fall solely on the shoulders of the military judge. At the conclusion of his inquiry into the providence of the guilty plea to the specification under Charge II, he asked counsel if either desired further questioning. Both counsel said they did not. Such a reply is all too common in cases where the issue before us is the providence of the plea. Trial counsel, in particular, should be ever vigilant during the plea providence inquiry and assist the military judge by suggesting areas of further inquiry concerning the elements of the offense or potential defenses.

Conclusion

Accordingly, the findings to Charge II and its specification are set aside. The remaining findings are affirmed….


Notes and questions on United States v. Coffman

  1. The kind of problem that arises in Coffman is often called a “mistake of fact.” When a defendant is mistaken about some key fact, will that mistake provide a defense to criminal liability? As is often true in law, it depends. Whether a defendant’s mistake about a factual issue is a defense to a criminal charge is a question of the mens rea requirement of the charged offense. Does larceny, as defined in the Code of Military Justice, require the defendant to know that the property he took belonged to another specific person (and was not abandoned)? Because the military court interprets the applicable statute to require knowledge that the property is owned by someone, the defendant’s mistaken belief that this property had been abandoned is relevant to his criminal liability. Compare to Morisette v. United States in Chapter Two.
  2. Looking at the larceny statute in the military code, can you identify other mistakes that would be relevant to a determination of guilt? What if the defendant knew that the gear belonged to a Marine, but mistakenly thought it belonged to one of his close friends in the platoon who “wouldn’t make a big deal” if the defendant took it? What if the defendant mistakenly believed the gear was worth less than $100, and mistakenly believed that the military larceny statute applied only to property worth over $100? A mistaken belief about what the statute criminalized would be characterized as a “mistake of law” rather than a mistake of fact. And whether mistakes of law matter to liability is again a question about the mens rea requirements of a particular statute. The key question is whether an accurate understanding of the applicable law is an element of the charged offense. Most statutes do not require knowledge of the law as an element, so it is usually true that “ignorance of the law is no excuse,” or not a valid defense. For more on mistakes of law and the rare circumstances in which a mistake of law can serve as a valid defense, see People v. Marrero in Chapter Seven.
  3. For our purposes, Coffman is useful not only to illustrate principles of mistake, but also as a rare instance of close judicial review of a guilty plea. The “providence inquiry” described by the appellate court is essentially a plea hearing, in which a military judge questions the defendant to be sure there is an adequate factual basis for the guilty plea. The appellate opinion that you’ve read then reviews (and reverses) the first judge’s finding that the plea was supported by fact, or “provident.” The basic principle that a guilty plea must have an adequate factual basis applies in state and federal courts as well as military ones. However, state and federal courts rarely scrutinize guilty pleas closely to ensure compliance with this requirement. The vast majority of guilty pleas (and the vast majority of all convictions in state and federal court) are not subject to any appellate review at all. Indeed, as discussed in Chapter Four, civilian courts sometimes accept “fictional pleas,” or guilty pleas to charges that could not possibly be proven given the available evidence. What factors might distinguish military courts from civilian ones, and lead to closer scrutiny of pleas in the military judicial system?

Burglary and Trespass

New Mexico Stat. Ann. 30-16-3. Burglary.

Burglary consists of the unauthorized entry of any vehicle, watercraft, aircraft, dwelling or other structure, movable or immovable, with the intent to commit any felony or theft therein.

A. Any person who, without authorization, enters a dwelling house with intent to commit any felony or theft therein is guilty of a third degree felony.

B. Any person who, without authorization, enters any vehicle, watercraft, aircraft or other structure, movable or immovable, with intent to commit any felony or theft therein is guilty of a fourth degree felony.

New Mexico Stat. Ann. 30-14-8. Breaking and entering.

A. Breaking and entering consists of the unauthorized entry of any vehicle, watercraft, aircraft, dwelling or other structure, movable or immovable, where entry is obtained by fraud or deception, or by the breaking or dismantling of any part of the vehicle, watercraft, aircraft, dwelling or other structure, or by the breaking or dismantling of any device used to secure the vehicle, watercraft, aircraft, dwelling or other structure.

B. Whoever commits breaking and entering is guilty of a fourth degree felony.

New Mexico Stat. Ann. 30-165. Possession of burglary tools.

Possession of burglary tools consists of having in the person’s possession a device or instrumentality designed or commonly used for the commission of burglary and under circumstances evincing an intent to use the same in the commission of burglary.

Whoever commits possession of burglary tools is guilty of a fourth degree felony.


STATE of New Mexico, Plaintiff-Appellee

v.

Franklin D. BEGAYE, Defendant-Appellant

Court of Appeals of New Mexico
505 P.3d 855

March 30, 2021

HANISEE, Chief Judge.

Defendant Franklin Begaye … was arrested on February 28, 2017, following a report of a break-in at Ram Signs, a business in Farmington, New Mexico. Testimony established that around 8:00 p.m. that night, Ram Signs co-owner Michael Mordecki heard a loud bang coming from the front of the building. Soon thereafter, Mr. Mordecki discovered that the front window had been smashed in and called the police. Officer Justin Nichols arrived [and] observed a broken window, an overturned cash box, and disarray around an employee’s desk. Nothing had been taken by the intruder, but the front office area had been rifled through….

Security footage provided by Monica Mordecki, also a co-owner of Ram Signs, revealed that the suspect was a male wearing light shoes, dark pants, and a dark jacket over a light hoodie. In searching nearby areas, Officer Nichols observed Defendant, who matched the description of the individual in the video, walking along Farmington’s main street, and upon approach, Officer Nichols saw what appeared to be shards of glass on Defendant’s jacket…. Officer Nichols detained and searched Defendant, finding a pair of black mechanic’s gloves, and a small red flathead screwdriver in the front pocket of Defendant’s pants….

Defendant was charged with fourth degree felony offenses of non-residential burglary, breaking and entering, and possession of burglary tools. At Defendant’s jury trial, the State presented testimony from, among other witnesses, Mr. and Mrs. Mordecki and Officer Nichols. The State also played the security camera footage, presented photographs of the scene, and admitted the clothing, boots, gloves, and screwdriver that Officer Nichols collected from Defendant on the night of the incident. Defendant was convicted on all charges. This appeal followed.

Defendant argues that his convictions for burglary and breaking and entering violate his right to be free from double jeopardy because both convictions are premised on the same act of a single unauthorized entry…. Double jeopardy protects defendants from receiving multiple punishments for the same offense.

Here, Defendant raises a double-description double jeopardy claim, “in which a single act results in multiple charges under different criminal statutes[.]” State v. Bernal (2006). “In analyzing double-description challenges, we employ [a] two-part test, … in which we examine: (1) whether the conduct is unitary, and, if so, (2) whether the Legislature intended to punish the offenses separately.” … “Only if the first part of the test is answered in the affirmative, and the second in the negative, will the double jeopardy clause prohibit multiple punishment in the same trial.” State v. Silvas (2015). Here, the State does not dispute Defendant’s contention that the conduct—the single unauthorized entry—was unitary. Accordingly, we consider the first part of the test to be satisfied…

Where, as here, Defendant’s conduct is unitary, we next analyze legislative intent, looking first to the language of the statutes. “Absent a clear intent for multiple punishments, we apply” [the test from Blockburger v. United States, 284 U.S. 299 (1932).] Blockburger provides that “the test to be applied to determine whether there are two offenses or only one, is whether each provision requires proof of a fact which the other does not.” “If one statute requires proof of a fact that the other does not, then the Legislature is presumed to have intended a separate punishment for each statute without offending principles of double jeopardy.” Silvas. “That presumption, however, is not conclusive and it may be overcome by other indicia of legislative intent.”

Since its adoption, the New Mexico Supreme Court has modified the Blockburger test, clarifying that application of the test “should not be so mechanical that it is enough for two statutes to have different elements.” When discerning legislative intent for the purpose of the modified Blockburger test, we may look to the “language, structure, history, and purpose” of the relevant statutes. “If the statutes can be violated in more than one way, by alternative conduct, the modified Blockburger analysis demands that we compare the elements of the offense, looking at the [s]tate’s legal theory of how the statutes were violated.” We may ascertain the state’s legal theory “by examining the charging documents and the jury instructions given in the case.”

Here, Defendant argues that the modified Blockburger test should apply to [his] double jeopardy claim. Defendant contends that within a modified analysis and under the State’s legal theory of the case, breaking and entering was subsumed within the burglary conviction, therefore, double jeopardy bars his conviction under the breaking and entering statute. Defendant further claims, in the alternative, that even if the elements of each statute are distinct, other indicia of legislative intent make clear that the Legislature did not intend to permit separate convictions under both the burglary and the breaking and entering statutes based on a single unauthorized entry. The State argues, in turn, that under either a strict or modified Blockburger test, Defendant’s convictions are not barred by double jeopardy because both offenses require proof of an element the other does not and the Legislature intended to permit separate convictions under the two statutes.

While there is no stated intent that the burglary and breaking and entering statutes allow for multiple punishments, we can presume the Legislature intended to allow separate punishment under the statutes because each provision requires proof of a factual element that the other does not. Section 30-16-3, prohibiting non-residential burglary, reads in pertinent part, “[b]urglary consists of the unauthorized entry of any … dwelling or other structure, movable or immovable, with the intent to commit any felony or theft therein.” Meanwhile, Section 30-14-8(A) prohibits breaking and entering and reads, in pertinent part, “[b]reaking and entering consists of the unauthorized entry of any … dwelling or other structure, movable or immovable, where entry is obtained by fraud or deception, or by the breaking or dismantling of any part of the … dwelling or other structure[.]” While both offenses require an unauthorized entry into a dwelling, the burglary statute requires a defendant to have a specific intent “to commit any felony or theft therein.” Further, the breaking and entering statute requires the unauthorized entry to be effectuated by a specified means, which the burglary statute does not. Therefore, under the Blockburger strict elements test, both offenses require proof of an element the other does not, and we can infer therefrom that the Legislature intended to authorize separate punishments under the burglary and breaking and entering statutes.

This inference, however, is not conclusive … [and] we apply the modified Blockburger test to examine other indicia of legislative intent. See State v. Ramirez (2016) (explaining that “[w]hen applying Blockburger to statutes that are vague and unspecific or written with many alternatives, we look to the charging documents and jury instructions to identify the specific criminal causes of action for which the defendant was convicted” and to determine whether the Legislature intended to allow separate punishments under multiple statutes).

Although we recognize that the purpose of “New Mexico’s breaking[ ] and[ ] entering statute is itself grounded in common law burglary[,]” each statute presents distinct objectives that we rely on to guide our analysis. To reiterate, breaking and entering requires an unauthorized means of entry, such as an actual “breaking.” In State v. Sorrelhorse (2011), we held that the offense of criminal damage to property was a lesser included offense of breaking and entering because both offenses require actual property damage. Sorrelhorse indicates that, where entry is obtained by breaking or dismantling physical property, the evident purpose of the breaking and entering statute is to punish unauthorized entry accomplished by physical damage to property.

In comparison, while the burglary statute is likewise intended to safeguard possessory property interests, the evolution of common law burglary in New Mexico leads us to believe that the Legislature intended to authorize separate punishments under the statutes. At common law, “[b]urglary consisted of breaking and entering a dwelling of another in the night time with the intent to commit a felony.” Initially, the crime required some physical act or element of force but did not specifically require damage to property. However, as the common law developed, the “breaking” component of common law burglary could be satisfied by a constructive breaking and did not necessarily require a physical act. For example, this Court held that “entry by fraud, deceit, or pretense was sufficient to constitute the ‘unauthorized entry’ requirement, which had been adopted by the New Mexico Legislature instead of the common law requirement of ‘breaking.’ ” Therefore, we conclude the purpose of the breaking and entering statute is sufficiently distinct from the purpose of the burglary statute. The crime of burglary punishes the broader criminal conduct of any unauthorized entry when there is specific criminal intent.

Having concluded that the Legislature intended to allow separate punishments under the two statutes, we turn next to the State’s theory of the case. A comparison of the instructions tendered to the jury for the two offenses establishes that the breaking and entering charge was not subsumed into the burglary charge. To convict Defendant of breaking and entering, the jury was required to find, in pertinent part, that (1) “[D]efendant entered a structure without permission”; and (2) “[t]he entry was obtained by the breaking of a window[.]” Meanwhile, a guilty verdict on the burglary charge required the jury to find, in pertinent part, that Defendant (1) “entered a structure without authorization[,]” and did so (2) “with the intent to commit a theft when inside.”

Although it agrees on appeal that Defendant’s entrance through the window of Ram Signs constituted unitary conduct for the purposes of both statutes, at trial the State did not suggest that the jury rely on the unauthorized entrance as the sole basis for conviction of each crime. Here, the crucial distinction in the two crimes is that the unauthorized entrance required by the burglary charge jury instruction also included the specific intent “to commit a theft when inside.” Hence, the State’s theory of the case for burglary required the jury to find something more than what was required for breaking and entering. Similarly, although the unauthorized entrance through the broken window was a common element of both charges, to convict Defendant of breaking and entering, the jury had to find that the unauthorized entrance was effectuated by breaking the window. That additional element—one that was not required by the burglary instruction—establishes that Defendant’s conviction for breaking and entering could not have been subsumed within the aggravated burglary conviction.

The charging documents specifically relied on the “breaking or dismantling” component of the breaking and entering statute in charging Defendant with breaking and entering, and relied on the “intent to commit a felony or theft therein” component of the burglary statute in charging Defendant with burglary. As such, the State’s theory of the case regarding the conduct required by the two charges was adequately distinguishable and not solely premised on the unitary conduct. Therefore, we hold that Defendant’s convictions for breaking and entering and aggravated burglary did not offend his right to be free from double jeopardy….


Notes and questions on State v. Begaye

  1. In addition to the burglary and breaking and entering charges, Franklin Begaye was also charged with possession of burglary tools—namely, gloves and a screwdriver, which were found in his pockets when he was arrested. The possession of burglary tools charge required proof that the items were commonly used in burglaries or that they had in fact been used in a specific burglary. In a portion of the opinion omitted here, the New Mexico court found that there was insufficient evidence that Begaye had actually used gloves or a screwdriver in the break-in at Ram Signs. Thus the court did not have to consider whether a defendant could be convicted of separate offenses of burglary and possession of burglary tools. Can you imagine why a legislature might want to criminalize the possession of burglary tools as a separate offense? Do you think the legislature likely intended to punish burglary, and possession of burglary tools, with separate punishments?
  2. Begaye involves a very common situation: multiple criminal statutes could potentially apply to a particular act by a defendant. You have previously encountered this scenario with State v. Cissell in Chapter Three. In that case, the defendant objected because he was convicted and sentenced under a statute with a more severe penalty when another statute, with a less severe penalty, was equally applicable to his conduct. Cissell argued that to charge him with the more severe offense violated equal protection and due process, but the Wisconsin court rejected his claim. Notice that Begaye’s claim here is a little bit different. Begaye raised a double jeopardy challenge, arguing that to punish him for both burglary and breaking and entering is to punish him twice for the same conduct. The Double Jeopardy Clause of the Fifth Amendment provides, “nor shall any person be subject for the same offense to be twice put in jeopardy of life or limb.” Whether the Double Jeopardy Clause bars multiple punishments depends on whether the punishments are for “the same offense,” and the analysis of that question can be complex. To evaluate such challenges, most states follow Blockburger v. United States, which is explained and discussed in this case.
  3. The facts of Begaye probably coincide with a fairly common perception of the crime of burglary: entry with probable intent to steal. Notably, though, burglary (as defined at common law, and also as defined in most modern statutes) does not actually require any intent to take property or otherwise commit a property offense rather than some other kind of crime. As the court notes, common law burglary was “breaking and entering a dwelling of another in the night time with intent to commit a felony.” Any felony would suffice, so a person who entered another’s dwelling with intent to kill the resident would be guilty of burglary. Modern statutes typically define burglary with an equivalent or similar mens rea requirement – intent to commit a felony or other crime inside the place burglarized. But many jurisdictions have eliminated other aspects of common law burglary, such as the requirement of “breaking,” as the Begaye court mentions and as discussed further in the next note. Modern burglary statutes also often omit any requirement that the entry be into a dwelling or that it take place at night.
  4. Again, the Begaye court describes common law burglary as “breaking and entering a dwelling of another in the night time with intent to commit a felony.” What is a “breaking,” as that term was understood at common law? Must a lock, a window, or some other part of the property actually get broken? Early judicial accounts of burglary required some act of physical force to enter the property, though this “breaking” by force did not need to cause lasting damage to the property. For example, many courts held that opening an unlocked but closed door or window was a sufficient “breaking” for the crime of burglary, but simply walking through an open door was not breaking. “As the common law developed,” the Begaye court tells us, “the ‘breaking’ component of common law burglary could be satisfied by a constructive breaking and did not necessarily require a physical act.” A person who tricked a homeowner into letting him in – pretending to be a city inspector, for example – could be guilty of burglary by “constructive” breaking.
  5. The usual mens rea of burglary – intent to commit a felony – is often described as a “specific intent” requirement. As discussed above, larceny is also often described as a “specific intent” crime, given the typical requirement of intent to permanently deprive the owner of the property. Courts often distinguish between “specific intent” and “general intent” crimes. These terms developed at common law and do not have a single uniform definition, but you should know how the terms are most commonly used. Usually, a description of a crime as a “specific intent” crime means that the crime is defined to include some mental state requirement beyond the defendant’s mental state regarding the actus reus of the offense. In contrast, the classification of an offense as one of “general intent” usually means that the only mens rea requirement is one related to the defendant’s mental state regarding the conduct elements of the offense. This explanation will probably be hard to grapple in the abstract, so an example is useful. In Begaye, the breaking and entering statute would probably be treated as a “general intent” offense. Notice that the New Mexico breaking and entering statute does not actually specify a mental state requirement at all. But given courts’ usual presumptions against strict liability, and given the mens rea default rules discussed in Chapter Two, a court would probably require that the defendant commit the actus reus of the offense – unauthorized entry – with at least knowledge or recklessness. The prosecution thus must establish something about the defendant’s mental state, but only the defendant’s mental state toward the actus reus of the offense: the defendant knew she was entering without authorization, or the defendant was at least reckless with regard to whether she was entering without authorization. Contrast that “general intent” with burglary, which requires evidence of a further mental state – namely, a plan to do something specific (commit a crime) once inside the place entered. In rejecting the defendant’s double jeopardy claim, one factor emphasized by the Begaye court is that burglary is a “specific intent” crime while breaking and entering is not.
  6. Given that burglary is entry with intent to commit any felony, prosecutors have often charged this offense in the context of domestic violence, when a person enters a residence in violation of a protective order or with intent to harm someone inside in the resident. Burglary statutes have even been used to prosecute persons for entering what is technically their own legal residence. In that context, burglary statutes sometimes come into conflict with “anti-ousting” statutes which provide that a husband and wife cannot exclude one another from their shared dwelling. Consider State v. Lilly, below.

Ohio R.C. 2911.12. Burglary; trespass in a habitation[ ]

(A) No person, by force, stealth, or deception, shall do any of the following:

(1) Trespass in an occupied structure or in a separately secured or separately occupied portion of an occupied structure, when another person other than an accomplice of the offender is present, with purpose to commit in the structure or in the separately secured or separately occupied portion of the structure any criminal offense;

(2) Trespass in an occupied structure or in a separately secured or separately occupied portion of an occupied structure that is a permanent or temporary habitation of any person when any person other than an accomplice of the offender is present or likely to be present, with purpose to commit in the habitation any criminal offense;

(3) Trespass in an occupied structure or in a separately secured or separately occupied portion of an occupied structure, with purpose to commit in the structure or separately secured or separately occupied portion of the structure any criminal offense.

(B) No person, by force, stealth, or deception, shall trespass in a permanent or temporary habitation of any person when any person other than an accomplice of the offender is present or likely to be present.

(D) Whoever violates division (A) of this section is guilty of burglary. A violation of division (A)(1) or (2) of this section is a felony of the second degree. A violation of division (A)(3) of this section is a felony of the third degree.

(E) Whoever violates division (B) of this section is guilty of trespass in a habitation when a person is present or likely to be present, a felony of the fourth degree.

Ohio R.C. 2911.21. Criminal trespass

(A) No person, without privilege to do so, shall do any of the following:

(1) Knowingly enter or remain on the land or premises of another…

Ohio R.C. 3103.04. Interest in the property of the other

Neither husband nor wife has any interest in the property of the other, except [as provided by a statutory duty to support one’s spouse], the right to dower, and the right to remain in the mansion house after the death of either. Neither can be excluded from the other’s dwelling, except upon a decree or order of injunction made by a court of competent jurisdiction.


The STATE of Ohio, Appellant

v.

Harold Dean LILLY, Appellee

Supreme Court of Ohio
87 Ohio St.3d 97

Decided Oct. 20, 1999

[Syllabus by the Court:] On February 5, 1997, Harold Dean Lilly, Jr., defendant-appellee, was indicted on nineteen criminal counts: twelve counts of rape, two counts of attempt to commit rape, three counts of possessing criminal tools, one count of kidnapping, and one count of burglary. All offenses were alleged to have been against his estranged wife, Jacqueline K. Lilly. Count nineteen of the indictment [charging burglary] stated that on or about January 26, 1997, defendant had trespassed in Jacqueline Lilly’s residence when she was present or likely to have been present, with the purpose of committing a criminal offense therein. Defendant pled not guilty on February 11, 1997 to all of the charges.

Defendant and Jacqueline K. Lilly (“Mrs. Lilly”) married in August 1988. They separated in early 1996 and got back together in September of that year. In November 1996, they separated again… [In January 1997] Mrs. Lilly … leased an apartment in West Carrollton, Ohio…. Defendant moved in with his own mother after the couple separated.

Mrs. Lilly testified that on January 26, 1997, she and the defendant spent the morning and afternoon together doing various errands. The defendant repeatedly asked Mrs. Lilly if they could watch the Super Bowl that evening together, but she declined. Mrs. Lilly testified that over the course of the evening, defendant asked her to have sex with him and she asked him to leave. She told the jury that defendant became angry, slapped her repeatedly, and burned her with a cigarette. She further explained that, to avoid further harm, she engaged in various sexual acts with defendant, which Mrs. Lilly testified were against her will.

Mrs. Lilly testified that later in the evening, the defendant drove her to two bars. At the 1470 Club, in Kettering, Ohio, Mrs. Lilly quietly asked one of the bar employees to call the police. After defendant followed her into the women’s restroom at the bar, one of the bar’s security guards went into the restroom to check on Mrs. Lilly. The security guard told defendant that he wanted to speak to Mrs. Lilly alone and defendant refused. The security guard pushed defendant out of the way while Mrs. Lilly and a female bar employee ran into the back office and locked the door. After the defendant’s attempts to kick the door in were unsuccessful, he fled.

Mrs. Lilly was taken to the hospital to be examined and then to the police station to be interviewed… Police officers then took her to her apartment to get some clothing and personal items in order for her to stay in a shelter. At her apartment, Mrs. Lilly discovered that her purse was missing and about six pairs of her jeans had been ripped up. Officers noticed that the attic cover was open [and] smelled fresh cigarette smoke.

After Mrs. Lilly had collected her belongings and was ready to get in her car, she discovered that her automatic garage door opener was missing from her car. She tried to start her car, and when it would not start, officers investigated and found that the car’s spark plug wires had been detached. In addition, Mrs. Lilly noticed a pair of defendant’s gym shoes that were not there previously. At approximately 8:00 a.m. on January 27, officers drove Mrs. Lilly to a shelter.

Detective Mark Allison testified that on the afternoon of January 27, he informed defendant that a warrant had been issued for his arrest. The next day, Detective Allison interviewed defendant… Defendant admitted to the officers that he drove back to Mrs. Lilly’s apartment in the early morning of January 27…. Defendant stated that he had left the door unlocked prior to leaving with Mrs. Lilly earlier in the evening so he could get back in. Defendant told the detectives that he ripped up several pairs of Mrs. Lilly’s jeans, yanked the spark plug wires on her car, and took her purse. Defendant stated that he arrived at the apartment around 12:30 a.m. on January 27, after leaving the bar and was there until 12:00 p.m. that day. Defendant admitted that he was hiding at the apartment when police searched it.

At trial, Mrs. Lilly testified that the lease for her apartment was in her name and the defendant did not have a key. Mrs. Lilly testified that defendant did not contribute money for her apartment. She further testified that defendant knew that it was her place.

During the trial, the state withdrew one count of rape and one count of attempted rape. The jury returned a verdict of guilty on the burglary charge and not guilty on the remaining charges. Defendant appealed his burglary conviction, and the Montgomery County Court of Appeals reversed the trial court’s conviction, finding that [a civil anti-ousting law] negated the state’s proof of the element of trespass as a matter of law.

Opinion

LUNDBERG STRATTON, J.

This case presents the court with the question of whether R.C. 3103.04 precludes prosecution of one spouse for burglary committed in the residence of the other spouse. For the reasons that follow, we hold that a spouse may be criminally liable for trespass and/or burglary in the dwelling of the other spouse who is exercising custody or control over that dwelling. R.C. 3103.04 is inapplicable in criminal cases.

In this case, the evidence showed that defendant entered by deception the separately leased property of his estranged spouse with intent to commit a crime. However, the court of appeals concluded that in the absence of a court order, R.C. 3103.04 prevented Mrs. Lilly from excluding defendant from her apartment and therefore the element of trespass could not be proven. Although the defendant did not raise this alleged R.C. 3103.04 privilege in the trial court, the court of appeals, nevertheless, found that it amounted to plain error. We disagree…

At common law, husband and wife were regarded as one. The legal existence of the wife during coverture was merged with that of her husband. As such, the wife was incapable of making contracts, of acquiring property, or of disposing of property without her husband’s consent. In pursuance of a more liberal policy in favor of the wife, statutes were passed across the country to relieve the married woman from the disabilities imposed upon her as a femme covert by the common law.

…[I]n 1887, the General Assembly enacted what is now R.C. 3103.04 to “define the rights and liabilities of husband and wife.” 84 Ohio Laws 132. The Act related both to the relationship between husband and wife and to the rights of each in the property of the other. [After some slight amendments over the years,] [t]he statute today is reflected in R.C. 3103.04:

“[Interest in the property of the other]

“Neither husband nor wife has any interest in the property of the other, except [specific statutory provisions]. Neither can be excluded from the other’s dwelling, except upon a decree or order of injunction made by a court of competent jurisdiction.”

… Notably, R.C. 3103.04 is situated in the domestic relations chapter of the Revised Code. Further, a review of the 1887 Act reveals that it primarily concerned property rights as they relate to domestic relations.

A review of other jurisdictions reveals seven other jurisdictions with a statute similar to R.C. 3103.04. … Significantly, we note that our review indicates that none of these jurisdictions applies this civil statute in criminal contexts.

Thus, we conclude that R.C. 3103.04 was intended to address property ownership rights of married persons, matters of a civil nature. Privileges of a husband and wife with respect to the property of the other were not meant to be enforced criminally and do not affect criminal liabilities. Because we find that the General Assembly never intended for R.C. 3103.04 to apply in criminal contexts, we must turn to the Criminal Code to address this issue.

The crime of burglary, with which defendant was charged, provides:

“(A) No person, by force, stealth, or deception, shall do any of the following:

“ * * *

“(2) Trespass in an occupied structure or in a separately secured or separately occupied portion of an occupied structure that is a permanent or temporary habitation of any person when any person other than an accomplice of the offender is present or likely to be present, with purpose to commit in the habitation any criminal offense * * *.” R.C. 2911.12(A)(2).

“Criminal trespass” is defined as:

“(A) No person, without privilege to do so, shall do any of the following:

“(1) Knowingly enter or remain on the land or premises of another.” R.C. 2911.21.

The law of burglary evolved out of a desire to protect the habitation. Because intrusions into the habitation are dangerous to occupants, “the offense is viewed as serious, because of the higher risk of personal harm involved in maliciously breaking and entering an occupied, as opposed to an unoccupied, structure.” 1974 Legislative Service Commission Comment to R.C. 2911.12.

Because the purpose of burglary law is to protect the dweller, we hold that custody and control, rather than legal title, is dispositive. Thus, in Ohio, one can commit a trespass and burglary against property of which one is the legal owner if another has control or custody of that property.

A majority of other jurisdictions that have addressed this issue have found that the entry of an estranged spouse upon the property of the other spouse constitutes an unauthorized entry to support charges of trespass and burglary….

Civil, peaceful avenues of redress exist to enforce the rights of a person who believes he or she has been wrongfully excluded from certain property. There is no privilege to use force, stealth, or deception to regain possession. See R.C. 2911.21(C) (“It is no defense to a charge under this section that the offender was authorized to enter or remain on the land or premises involved, when such authorization was secured by deception.”).

In this case, there is no evidence that defendant had any right to custody or control of the leased property. The apartment was leased solely in Mrs. Lilly’s name. Defendant did not pay any part of the rent on Mrs. Lilly’s apartment. While defendant claims that he may have stayed at the apartment occasionally and performed maintenance tasks there for Mrs. Lilly, defendant never lived at the apartment, did not have a key to the apartment, and did not keep any of his belongings in the apartment. Accordingly, it was reasonable for the jury to find that when, without permission, defendant entered Mrs. Lilly’s apartment through a door he had previously by deception left unlocked, he trespassed. When he trespassed in Mrs. Lilly’s apartment for the purpose of committing a crime, i.e., theft of her purse and damage to her property, it was reasonable for the jury to conclude that defendant committed a burglary.

Thus, there was ample evidence at trial for the jury to have determined that the defendant trespassed in Mrs. Lilly’s dwelling and that he did so with the purpose or intent of committing a crime. Sufficiency of the evidence is considered in a light most favorable to the prosecution. As such, we find that there was sufficient evidence of burglary to sustain his conviction. Therefore, we reverse the judgment of the court of appeals and reinstate defendant’s conviction for burglary…


Notes and questions on State v. Lilly

  1. In Ohio, and in many other jurisdictions, the offense of burglary is now defined a trespass with the intent to commit a crime (other than the trespass itself) in the place being entered. (What crime did Lilly intend to commit in his wife’s apartment, according to this court?) But as you will see in Chapter Eight, doctrines of criminal attempt already impose criminal liability for those who intend to commit an offense and take steps toward the commission of that offense. Someone who enters a home with intent to steal from it is likely liable for attempted theft. Why, then, is it necessary to have burglary as a separate criminal offense? The drafters of the Model Penal Code considered this issue, but ultimately decided to keep a burglary offense as part of their recommended code. As the drafters explained,
    The critical issues to be confronted in the law of burglary are whether the crime has any place in a modern penal code and, if so, how it should be graded. The first question arises because of the development of the law of attempt. Traditionally, an independent substantive offense of burglary has been used to circumvent unwarranted limitations on liability for attempt. Under the Model Code, however, these defects have been corrected. It would be possible, therefore, to eliminate burglary as a separate offense and to treat the covered conduct as an attempt to commit the intended crime plus an offense of criminal trespass. Section 221.1 nevertheless continues burglary as an independent substantive offense carrying felony sanctions. In part, this solution reflects a deference to the momentum of historical tradition. More importantly, however, the maintenance of a crime of burglary reflects a considered judgment that especially severe sanctions are appropriate for criminal invasion of premises under circumstances likely to terrorize occupants.

    Model Penal Code 221.1, explanatory note.

  2. Note that Harold Lilly was indicted on nineteen counts, including twelve counts of rape, two counts of attempted rape, and a kidnapping charge. Note also that there were witnesses to some of Mr. Lilly’s conduct: security guards at a bar tried to protect Jacqueline Lilly from her husband, and a female bar employee fled to a back office with Jacqueline. But Harold Lilly was acquitted of all charges except the burglary charge. We don’t have access to the jury’s deliberations, but can you imagine possible reasons that the jury might have convicted only on the burglary charge and acquitted on all others?
  3. Does the Lilly court rely on a sharp dichotomy between civil law (such as the anti-ousting statute) and criminal law? Or does the court’s decision tend to collapse the distinction between civil and criminal law? Professor Jeannie Suk has characterized responses to domestic violence like the one we see in Lilly as a form of “state-imposed de facto divorce.” She writes,

    …The Ohio Supreme Court took the position that the criminal law would ignore the anti-ousting statute. The anti-ousting provision “was intended to address property ownership rights of married persons, matters of a civil nature. Privileges of a husband and wife with respect to the property of the other were not meant to be enforced criminally and do not affect criminal liabilities.” Because the anti-ousting statute regulated in the domains of property and family relations, it simply did not apply in a criminal case…

    What is notable here is the purportedly easy division of the world into criminal and civil spheres of regulation. If applied, the anti-ousting statute would have directly conflicted with the spousal burglary conviction. According to the theory the court adopted, criminal and civil spheres were mutually exclusive and thus the civil anti-ousting statute, which regulated property interests, could have no effect on the criminal law question of burglary.

    But the crime of burglary does not operate apart from a property regime. The court’s assertion that property law and criminal law represented wholly separate spheres deviated from the common law relation between burglary and property law. Classically, burglary law was dependent upon the underlying allocation of property rights. The criminal law question of whether a person committed burglary depended on property law for its application. The underlying property arrangement determined whether his entry was burglarious.

    The Lilly court indicated its intention to treat the criminal and civil spheres as wholly separate for purposes of this case. But by declining to apply the anti-ousting statute in a burglary case, the court was actually allowing criminal law, as [domestic violence] policy, to trump the law of property. The effect was to reallocate property rights between spouses such that burglary would [apply.]

    … What we see here is a reversal of the dependence of burglary law on the law of property. Whereas traditionally, burglary depended on the prior allocation of possessory rights determined by property law, we now see the criminal law subordinating property law to its interests, in effect reallocating private rights. While property law had previously set the conditions for burglary, criminal law now takes precedence in defining property rights in this DV context. … Even as it claims to treat civil interests as a separate sphere, the criminal law, through its coercive power and its claim to the public interest, has an unmatched capacity to reorganize private interests.

    Jeannie Suk, Criminal Law Comes Home, 116 Yale L.J. 2 (2006).

  4. Given the serious allegations against Harold Lilly, and given the jury’s failure to convict on any of the rape and kidnapping charges, one might be relieved that burglary was available as a proxy charge in this case. Keep in mind that the same broad criminal laws that will make it easier to convict Lilly of something will make it easier to convict other defendants, too. Trespass is even easier to prosecute than burglary, and in some contexts civil rights advocates have contended that trespass laws are used to harass and oppress persons of color. In 2012, a number of civil rights organizations filed a federal lawsuit to challenge “Operation Clean Halls,” an NYPD program in public housing projects and other large residential buildings. Under the program, police patrolled the buildings and subjected many occupants to stops and frisks, often filing trespass charges against those who could not prove residence in the building. “Many tenants who live in Clean Halls buildings are restricted in their ability to maintain familial ties and friendships due to the use of aggressive police tactics in their homes,” the New York Civil Liberties Union explained in a public statement about its lawsuit. “The program is part of a citywide practice of suspicionless police stops and arrests that primarily impact communities of color.” The lawsuit, Ligon v. City of New York, eventually settled (along with the more widely publicized lawsuit challenging NYPD stop-and-frisk practices, Floyd v. City of New York). But Operation Clean Halls continued in a modified form under a new name, the Trespass Affidavit Program, until it was shut down in the fall of 2020. Other cities continue to operate Trespass Affidavit Programs, under which building owners submit lists of building residents to law enforcement and invite law enforcement to patrol the buildings and arrest non-residents.

Check Your Understanding (5-2)

Robbery

N.J.S.A. 2C:15-1a(1)

a. Robbery defined. A person is guilty of robbery if, in the course of committing a theft, he:

(1) Inflicts bodily injury or uses force upon another; or

(2) Threatens another with or purposely puts him in fear of immediate bodily injury; or

(3) Commits or threatens immediately to commit any crime of the first or second degree.

An act shall be deemed to be included in the phrase “in the course of committing a theft” if it occurs in an attempt to commit theft or in immediate flight after the attempt or commission.

b. Grading. Robbery is a crime of the second degree, except that it is a crime of the first degree if in the course of committing the theft the actor attempts to kill anyone, or purposely inflicts or attempts to inflict serious bodily injury, or is armed with, or uses or threatens the immediate use of a deadly weapon.


STATE of New Jersey

v.

Francisco SEIN

Supreme Court of New Jersey
590 A.2d 665

Decided May 21, 1991

CLIFFORD, J.

The narrow issue on this appeal … is whether the sudden snatching of a purse from the grasp of its owner involves enough force to elevate the offense from theft from the person to robbery as defined by N.J.S.A. 2C:15-1a(1)….

-A-

On August 27, 1986, Edythe Williams cashed her unemployment check at Proper Check Cashing, a concession located in the Woolworth store on Main Street in Paterson. Mrs. Williams placed the proceeds in a zipped compartment in the strapless, clutch-type purse that she carried under her arm. After purchasing a notebook in Woolworth’s, she left the store and headed for her car, which she had parked a couple of blocks away.

Mrs. Williams arrived at her car intending to drop off the notebook and continue shopping in the area. She went to the passenger side and put her key in the lock, all the while carrying the purse under her right arm. As Mrs. Williams stood in the street, defendant, Francisco Sein, walked up and stood close beside her on her left. Mrs. Williams turned to face the man, thinking he had approached to ask a question, but defendant said nothing. Instead, “he reached across [her] and just slid [her] pocketbook–which wasn’t very hard to do–from under [her] arm and took off,” running toward Main Street. There was no evidence that defendant used any force other than that required to slide the purse from beneath Mrs. Williams’ arm.

The police apprehended defendant, who was subsequently indicted for robbery… At trial, defendant moved at the conclusion of the State’s case for a judgment of acquittal in respect of the robbery charge, contending that the case should proceed only on the lesser-included offense of theft from the person, defined by N.J.S.A. 2C:20-3a as the “unlawful[ ] tak[ing], or exercis[ing of] unlawful control over, movable property of another with purpose to deprive him thereof.” The crux of defendant’s argument was that there was no evidence in the record that the taking of Mrs. Williams’ purse was accompanied by the use of force against her person, a requirement for conviction under N.J.S.A. 2C:15-1a(1). The State, on the other hand, urged that a judgment of acquittal would be improper because the Legislature intended that the force used to remove the purse from the victim was sufficient to elevate the unlawful taking to a robbery. The trial court denied defendant’s motion, and the jury subsequently found defendant guilty of second-degree robbery.

On appeal, defendant contended that the trial court had erred by submitting the second-degree-robbery charge to the jury because there was no evidence that defendant had used force on Mrs. Williams in the course of the purse-snatching. The Appellate Division agreed… The court reversed the robbery conviction and remanded for the entry of a judgment of conviction for theft and for resentencing for that offense.

Before us, the State argues that the Appellate Division’s construction of the “uses force upon another” language in the robbery statute, N.J.S.A. 2C:15-1a(1), both misconstrues the plain meaning of the statute and contravenes the relevant legislative intent. According to the State, the Appellate Division’s standard will “change the focus of a robbery committed through the use of force from the conduct of the perpetrator to the nature of the property that he stole * * * and the particular characteristics of the victim as well as the victim’s actions.” In addition, the State submits that the standard established by the Appellate Division to determine the amount of force necessary to effect a robbery is “inexact and unworkable,” and that therefore jurors will be required to use concepts founded in the science of physics to determine whether more force was used than that quantum necessary merely to remove the object.

-B-

Cases involving “snatching” have required courts to determine where to draw the line between robbery and the lesser offense of larceny from the person. A certain amount of “force” is necessary to take property from the person of another, but whether the amount necessary merely to accomplish that taking is sufficient to warrant the more serious penalties associated with robbery has vexed those courts that have considered the question.

Some jurisdictions have construed the term “force” as used in the state’s robbery statute to mean mere physical force or energy, while others have rejected hypertechnical distinctions in favor of a view that acknowledges that snatching an object from the grasp of the owner increases the risk of danger to the victim and justifies enhanced punishment. Those jurisdictions implicitly recognize that victims do not turn over their property willingly, even if they do not resist or struggle with a thief. Thus, the amount of physical energy necessary to take the property is deemed sufficient to support a robbery conviction.

The predominant view, however, is that there is insufficient force to constitute robbery when the thief snatches property from the owner’s grasp so suddenly that the owner cannot offer any resistance to the taking. See W. LaFave & A. Scott, Criminal Law § 8.11(d), at 781 (2d ed.1986). This “majority rule” has been set forth in the following terms:

[A] simple snatching or sudden taking of property from the person of another does not of itself involve sufficient force to constitute robbery, though the act may be robbery where a struggle ensues, the victim is injured in the taking, or the property is so attached to the victim’s person or clothing as to create resistance to the taking.

People v. Patton (Ill. 1979).

The legislative history of New Jersey’s robbery statute, N.J.S.A. 2C:15-1, when read in the context of the Code Commentary on theft, reveals that our Legislature intended to adopt the majority rule.

-C-

At common law, robbery was defined in New Jersey as “the felonious taking of personal property from the person or custody of another by force or intimidation.” The pre-Code robbery statute, N.J.S.A. 2A:141-1, codified the common law.

The Appellate Division summarized the general state of the law of robbery under the pre-Code statute in State v. Culver (1970), where it stated:

N.J.S.A. 2A:141-1 provides that any person who forcibly takes from the person of another money or personal goods and chattels of any value whatever by violence or putting him in fear, is guilty of a high misdemeanor. Thus, force or intimidation is a necessary element of the crime and must precede or be concomitant with the taking. The property stolen need not have been in contact with the person from whom it was taken at the time it was stolen, and if taken by fear it must be the result of such demonstration or threat as to create reasonable apprehension on the part of the victim that, if the theft were resisted, force would be used. While a secret or sudden taking of property from the owner without putting him in fear and without open violence is deemed larceny, if there be a struggle to keep it or any violence or disruption, the taking is robbery.

The foregoing summary suggests that [before 1979] New Jersey followed the majority view as stated in People v. Patton, above.

In 1979, the Legislature revamped the criminal laws by enacting the New Jersey Code of Criminal Justice… Under [the Code’s robbery statute] as originally enacted, “[a] person [was] guilty of robbery if, in the course of committing a theft, he: (1) [i]nflicts bodily injury upon another; or (2) [t]hreatens another with or purposely puts him in fear of immediate bodily injury; or (3) [c]ommits or threatens immediately to commit any crime of the first or second degree.”

In 1981, however, the Legislature amended N.J.S.A. 2C:15-1a(1) to read that a person is guilty of robbery if in the course of committing a theft he “[i]nflicts bodily injury or uses force upon another.” L. 1981, c. 22, § 1 (emphasis added). The Legislature’s intention regarding the addition of the “or uses force” language is made clear by the following Statement of the Senate Judiciary Committee:

Senate Bill No. 885 amends N.J.S.A. 2C:15-1 to clarify that a person is guilty of robbery if he uses any force upon another in the course of committing a theft. Under present law only a person who inflicts bodily injury upon another in the course of committing a theft is guilty of robbery. Senate Bill 885 extends the definition of robbery to cover the so-called “blind-side” mugging. This occurs when a person commits an act of theft-for example a purse snatching-by approaching the victim from behind and using some degree of force to wrest the object of his theft from the victim. Often, however, no bodily injury is inflicted in these cases and therefore the offenses committed could be found to be theft rather than robbery.

[Statement of the Senate Judiciary Committee to Senate Bill 885 (Apr. 21, 1980) (emphasis added).]

The State contends that that Statement shows that the Legislature contemplated that a sudden, surprise snatching of property held in the grasp of another or in some way in contact with the person of another involves the use of force sufficient to elevate the taking to a robbery.

To the contrary, that the Legislature intended to broaden the concept of force beyond the pre-Code understanding of that term is not at all clear. The Senate Judiciary Committee Statement begins by saying the amendment is to “clarify that a person is guilty of robbery if he uses force upon another in the course of committing a theft.” (Emphasis added.) That suggests that the omission of the “or uses force” language in the Code as originally enacted in 1979 was an oversight. In amending N.J.S.A. 2C:15-1a(1), the Legislature merely intended to clarify that the type of force required to support a robbery conviction under the pre-Code statute still would be sufficient to elevate a theft to a robbery.

As we indicated earlier, “a simple snatching or sudden taking of property from the person of another does not of itself involve sufficient force to constitute robbery” under the pre-Code statute, and nothing in the Senate Judiciary Committee Statement undercuts that standard. Although the Committee Statement refers to a “purse snatching” as an example of the conduct the amendment was intended to cover, it goes on to state that snatchings rising to the level of robbery include only those that involve “some degree of force to wrest the object” from the victim. (Emphasis added.) To “wrest” is to “pull, force, or move by violent wringing or twisting movements.” Webster’s Third New International Dictionary 2640 (1971). The Legislature apparently determined that the violence associated with “wresting” is deserving of more severe punishment. It did not, however, intend to eliminate the requirement that robbery by use of force include force exerted “upon another.”

Moreover, the Commentary to the Code definition of “theft” strongly suggests that the Legislature did not intend that a surprise purse-snatching unaccompanied by injury, threat, struggle, or attempted resistance would constitute the crime of robbery. In discussing N.J.S.A. 2C:20-3 [New Jersey’s theft statute], which provides in pertinent part that “a person is guilty of theft if he unlawfully takes, or exercises unlawful control over, movable property of another with purpose to deprive him thereof,” the Legislature made clear the following:

The crime here defined may be committed in many ways, i.e., by a stranger acting by stealth or snatching from the presence or even the grasp of the owner or by a person entrusted with the property as agent, bailee, trustee, fiduciary or otherwise.

[II New Jersey Penal Code: Final Report of the New Jersey Criminal Law Revision Commission.]

The theft statute thus includes purse-snatchings from the grasp of an owner, while the robbery statute includes purse-snatchings that involve some degree of force to wrest the object from the victim. The only way to reconcile the two statutes is to hold that robbery requires more force than that necessary merely to snatch the object.

If the Legislature had intended that the amount of force necessary to snatch the object would be sufficient to constitute a robbery, it could have amended the theft statute to indicate that it includes only those snatchings in which the object of the theft is loose or can be cut loose, but not those in which the object must be removed from the victim. It did not do so.

The standard we adopt today continues the focus of a robbery on the conduct of the perpetrator rather than on the nature of the property stolen or the characteristics of the victim and his or her actions. Furthermore, we do not agree with the State’s contention that this standard is “inexact and unworkable.” If in fact jurors will henceforth be required to resort to concepts founded in the science of physics to determine whether more force was used than that quantum necessary merely to remove the object, that is hardly a dismaying by-product of a correct interpretation of the statute. Such concepts are used frequently by juries in their deliberations and are entirely within their ken.

-D-

… There is no indication that the Legislature intended to change the pre-Code rule that “a secret or sudden taking of property from the owner without putting him in fear and without open violence is deemed larceny, [but] if there be struggle to keep it or any violence or disruption, the taking is robbery.” To the extent that the robbery statute and the Senate Judiciary Committee Statement are burdened with ambiguity, as so persuasively argued by the Chief Justice in his dissent, that ambiguity surely cannot inure to the benefit of the State. “[P]enal statutes that are open to more than one reasonable construction must be construed strictly against the State.”

Because there is no evidence that defendant’s conduct involved the type of force sufficient to elevate the theft to a robbery under N.J.S.A. 2C:15-1a(1), the judgment of the Appellate Division is affirmed.

WILENTZ, C.J., dissenting.

I would hold that, under the statutory amendment, all purse snatchings are robberies, regardless of the amount of force used. I believe this construction of the amendment achieves the overriding goal of the Legislature: to deter and to prevent purse snatchings, not some kind, one kind, or a particular kind, but all purse snatchings.

I recognize the legitimacy of the majority’s decision. The issue of statutory interpretation facing the Court is difficult. But I cannot accept what I believe is an interpretation that falls far short of the legislative intent even in the face of the persuasive analysis in its support. A basic societal interest is left unprotected despite the Legislature’s clear intent to protect it. It is our interest in personal safety.

Determining the appropriate judicial role in construing statutes that do not accurately reflect the Legislature’s intent is sometimes problematic. That problem is magnified as the distance between the legislative intent and the Legislature’s language grows. Where, as here, a criminal statute is involved, the judiciary’s attempt to bridge the gap can conflict with notions of fairness and due process: it is the criminal statute that determines a crime—we do not jail people for violating legislative intent.

This case presents an extreme example of the problem. The Legislature clearly intended to make every purse snatching a robbery and just as clearly did not say so. The question is how far the judiciary should go to effect the legislative purpose despite the lack of legislative language.

…A literal application of the “uses force upon another” language to the offense of purse snatching suggests that force used solely upon a purse would be insufficient to elevate the crime from theft to robbery. Pursuant to such a construction, one who snatched a purse without touching the victim, even if he exerted a great deal of force in order to pull a purse tightly clutched under his victim’s arm, would not be guilty of robbery. A robbery would result only if he pulled the owner’s hand itself, pushed her arm to loosen the clutch, or in some way used force upon the person as well as on the purse. That reading of the statute, closely conforming to the statutory language, would remove a substantial portion of purse snatching offenses from the robbery statute, contrary to the Legislature’s intent… The majority and this dissent both recognize that the statute should not be interpreted so literally. We differ, however, in our reading of the legislative history concerning the nature and strength of the legislative purpose, and therefore differ concerning the appropriate statutory interpretation.

I disagree with the majority’s interpretation of the Judiciary Committee’s Statement…. The Statement … taken as a whole, clearly suggests that the Legislature (to the extent the Judiciary Committee is its surrogate) intended to make all purse snatchings robberies, as did the Governor when he signed the bill.

… Certainly, given the text of the statutory amendment and the less than perfect clarity of the Committee Statement, one cannot fault the majority for concluding that the nature of the force determines the quality of the offense. My own reading of the legislative intent arises from other sources as well, however, sources of which we can fairly take judicial notice. Those sources are the vast increase in muggings and purse snatchings that preceded the law and society’s acute concern over these crimes.

For more than a decade we have witnessed a seemingly unprecedented upsurge in crime. Prominent among these offenses have been “street crimes,” stranger-to-stranger offenses including ordinary muggings (usually thought to include some degree of violence or its threat) and “purse snatchings,” meaning just what it says, not necessarily implying any violence or force whatsoever. That no one could any longer safely walk the streets produced fear and fury on the part of society. Purse snatching was particularly loathsome, given its unpredictability, lack of warning, almost total randomness, and the fact that women were almost invariably the victims. Force and violence were often present and certainly they were major elements of society’s concern. But it was the offense itself, and its usual attributes, the insult and offense to the person, the potential danger, the helplessness, and the utter unredeemable ugliness of being “attacked”—for that is the universal perception–with or without force, by some amoral stranger who takes command of your belongings—it was the offense itself, purse snatching, regardless of its differing qualities, that was society’s concern. I find it most difficult to believe that either society or the Legislature ever intended to further penalize only “wresting” purse snatchings, but not the swift skillful removals that involve practically no force. Society and the Legislature condemn this offense regardless of whether the victim resists or whether because of “wresting,” she becomes immediately aware of the violation. This is not to say that the Legislature could not have selected the extent of force as a critical factor in elevating the crime from theft to robbery. Certainly, the dangers are increased when a thief uses excessive force, whether because of an utter disregard for the victim or because of the necessities caused by her resistance; and to the extent force bears on the victim’s “awareness,” with its resulting fear, that is one of the most offensive aspects of the crime. The Legislature did not, however, intend to define the crime by the variables of fear and force. Rather, it intended to define all purse snatchings as robberies.

… It seems most unlikely that the Legislature sought to transform a thief into a robber only if the thief encounters resistance while accomplishing his goal. Such a result amounts to equating the defendant’s blameworthiness with the victim’s reflexes. A thief who finds it necessary to tug the purse from under a woman’s arm or from her instinctively tightened grasp is deemed a robber, while one who slides the purse out and removes it, as in this case, remains merely a thief. Similarly, one who takes a purse suspended by a shoulder strap or chain while the woman’s arm is resting on the purse is a robber, while one who covertly lifts the purse without engaging her conscious or inertial resistance is but a thief.

…Deterrence of purse snatching is what the Legislature wanted to accomplish, and it would be most surprised to learn that all it had deterred was purse snatching involving wresting, and had left out the many purse snatchers who go out ready and willing to wrest, but who, because of some fortuitous circumstance, grab the purse without such consequence. The announcement it wanted to make to society is that all purse snatchings are robberies so that the offenders should know it when they go out in the night—and more and more frequently, even in the day—to prey on their helpless victims.

The Legislature knew what too many of us know—that it is not only the one on a motor bike who takes the tourist’s purse as he strolls on the street, not only the person on the down escalator who grabs the shoulder bag from the victim on the up escalator and dashes off into the crowd, but more so it knew what is not shown on the television ads—the street criminal who pulls your purse out from under your arm, from behind, or face-to-face, without noticeable force and without threat other than the perceived possibility that resistance may lead to injury or even death. In each case the victim does not resist, and the only force may be the lifting of the object, but surely this is the force that the Legislature had in mind.

This construction of the statute is, of course, not limited to purses or to women. A person commits a robbery whenever he or she unlawfully takes an article held by or within the possession of a person, or attached to the outside of a person’s garments. This definition encompasses any kind of article, not just a purse, and it would presumably extend to pickpocketing, although the legislative history, overwhelmingly indicating an intent to reach purse snatchers, provides no indication of an intent to reach the pickpocket. If, as is entirely possible, the Legislature did not intend so broad a scope for its new definition of robbery when applied to pickpocketing, legislative amendment may be needed. Distinctions exist between the two: purse snatching is usually accompanied by some degree of force and victim awareness, while pickpocketing is usually characterized by stealth, lack of force, and no victim awareness.

…I would reverse the judgment of the Appellate Division and reinstate the guilty verdict.


Notes and questions on State v. Sein

  1. As noted in the introduction to this chapter, robbery is often classified as a “violent crime” rather than a property offense. Robbery does involve the taking of property, but it typically requires the taking of property by a specific means: the use or threat of physical force. The exercise of force (or the threat to exercise it) is a frequent concern of criminal law, and the next chapter will examine crimes involving force or violence in much greater detail. Once recurring question will be the one raised here in Sein: what exactly does the word “force,” or the word “violence,” mean when it is used in a criminal statute? About a year after deciding State v. Sein, the New Jersey Supreme Court addressed the meaning of the term “force” again, this time in the context of sexual assault. You’ll that case, In re. M.T.S., in Chapter Six, and have an opportunity then to think more about whether the concept of force is or should be defined uniformly across criminal law.
  2. Between majority and dissent in Sein, who offers the more convincing interpretation of the state legislature’s intent with respect to this particular statute? Is it clear that Judges Clifford (for the majority) and Chief Judge Wilentz (dissenting) are trying to ascertain the legislature’s intent, as opposed to trying to vindicate their own instincts about whether purse snatchings should be criminalized as robberies?
  3. Consider carefully the end of the dissent, where Chief Judge Wilentz says that his interpretation of the statute is “not limited to purses or women.” If that is true, why wouldn’t pickpocketing become robbery, according to the dissent?
  4. Use robbery to practice your ability to analyze mental state requirements. What is the mens rea requirement of the New Jersey statute? (Remember, the absence of an explicit textual reference to mental states does not mean that a statute lacks a mens rea requirement!) Do you think robbery would be classified as a “specific intent” or “general intent” crime?

N.Y. Penal Law 160.00. Robbery; defined

Robbery is forcible stealing. A person forcibly steals property and commits robbery when, in the course of committing a larceny, he uses or threatens the immediate use of physical force upon another person for the purpose of:

    1. Preventing or overcoming resistance to the taking of the property or to the retention thereof immediately after the taking; or
    2. Compelling the owner of such property or another person to deliver up the property or to engage in other conduct which aids in the commission of the larceny.

The People of the State of New York

v.

Edward REID

 

The People of the State of New York

v.

Walter RIDDLES

Court of Appeals of New York
508 N.E.2d 661

Decided May 5, 1987

SIMONS, J.

The common issue presented by these two appeals is whether a good-faith claim of right, which negates larcenous intent in certain thefts, also negates the intent to commit robbery by a defendant who uses force to recover cash allegedly owed him. We hold that it does not….

I

Defendant Edward Reid was charged in a multicount indictment with felony murder, three counts of robbery in the first degree, one count of criminal possession of a weapon in the third degree and various other crimes. He was acquitted of the murder count but convicted of the robbery and possession counts. The additional charges were dismissed by the trial court.

The convictions stem from defendant’s forcible taking of money from three others. The evidence established that defendant and his stepbrother, Andre McLean, approached … three men … standing on a street corner in The Bronx. Defendant and McLean were holding pistols when defendant demanded that the three men hand over money “that belonged to him,” apparently referring to money owed him as the result of prior drug transactions. [Two men] gave defendant money but [a third, Donnie Peterson] responded that he had none and would have to go upstairs to his apartment to get some. As the men walked up the stairs, toward Peterson’s apartment, defendant “snatched” McLean’s pistol, placed it in his waistband and demanded that McLean turn over money he was holding for him. McLean handed defendant $300. A moment later, he rushed at defendant, a “shot went off” striking McLean and defendant fled. McLean subsequently died from a single gunshot wound to his chest.

Defendant Walter Riddles was indicted for robbery in the second degree and assault in the second degree. He was convicted after a bench trial of robbery in the third degree for forcibly taking money from Genevieve Bellamy on November 10, 1982.

Bellamy and defendant both testified at trial, each providing different descriptions of events. Bellamy maintained that while she was waiting for a taxi at a street corner in The Bronx, defendant, whom she did not know, drove up to the curb and asked for directions. According to Bellamy, when she leaned into defendant’s automobile to help him, defendant grabbed her, forced her into the car and demanded money from her. Bellamy stated she did not have any, but defendant struck her in the face, searched her pockets, and, upon discovering $50, took the money and ordered her out of the automobile.

Defendant disputed her story. He testified that he knew Bellamy prior to the incident and that she owed him $25. He stated that … on the evening of November 10 … she offered to pay him $15 toward her debt if he drove her downtown so she could pick up a package. Defendant maintained that he took Bellamy downtown, as she asked, but that she was unable to obtain her package so he drove her back uptown. Defendant testified that during the return trip, Bellamy again offered to pay him $15 toward her debt, but upon seeing her counting a large sum of money, he took the full amount she owed him, $25, and no more.

In pronouncing judgment, the court stated that it credited the portion of defendant’s testimony indicating that he had taken the money from Bellamy to satisfy a debt but the court held that because defendant used force he was nevertheless guilty of robbery.

II

A person “commits robbery when, in the course of committing a larceny, he uses or threatens the immediate use of physical force.” The larceny statute, in turn, provides that an assertion that “property was appropriated under a claim of right made in good faith” is a defense to larceny (see Penal Law § 155.15). Since a good-faith claim of right is a defense to larceny, and because robbery is defined as forcible larceny, defendants contend that claim of right is also a defense to robbery. They concede the culpability of their forcible conduct, but maintain that because they acted under a claim of right to recover their own property, they were not guilty of robbery, but only some lesser crime, such as assault or unlawful possession of a weapon.

Defendants’ general contention is not without support. Several jurisdictions have held that one who acts under a claim of right lacks the intent to steal and should not be convicted of robbery. That logic is tenable when a person seeks to recover a specific chattel: it is less so when asserted under the circumstances presented in these two cases: in Reid to recover the proceeds of crime, and in Riddles, to recover cash to satisfy a debt.

We have not had occasion to address the issue but the Appellate Divisions [lower NY courts] to which it has been presented have uniformly ruled that claim of right is not a defense to robbery. Their determinations have been based upon the interpretation of the applicable statutes and a policy decision to discourage self-help and they are consistent with what appears to be the emerging trend of similar appellate court decisions from other jurisdictions. For similar reasons, we conclude that the claim of right defense is not available in these cases. We need not decide the quite different question of whether an individual who uses force to recover a specific chattel which he owns may be convicted of robbery. It should be noted, however, that because taking property “from an owner thereof” is an element of robbery, a person who recovers property which is his own (as compared to the fungible cash taken to satisfy a claimed debt in the cases before us) may not be guilty of robbery.

The claim of right defense is found in the larceny article of the Penal Law, which provides that a good-faith claim of right is a defense to trespassory larceny or embezzlement. The defense does not apply to all forms of larceny. For example, extortion is a form of larceny, but the Legislature, consistent with a prior decision of this court, has not authorized a claim of right defense to extortion. The exception is significant for extortion entails the threat of actual or potential force or some form of coercion. Thus, the inference may reasonably be drawn that in failing to authorize a claim of right defense for extortion in Penal Law § 155.15 (1), and by failing to incorporate it in article 160 of the statute, which governs robbery, the Legislature recognized that an accused should not be permitted to invoke it in crimes involving force. We assume that if the Legislature intended to excuse forcible taking, it would have said so.

Our decision also rests upon policy considerations against expanding the area of permissible self-help. Manifestly, a larceny, in which the accused reacquires property belonging to him without using force, differs from a robbery in which the defendant obtains money allegedly owed to him by threatening or using force. “The former is an instance of mistake, not subjected to penal sanctions because the threat to private property is not so serious as to warrant intervention by the criminal law. The latter is a species of self help and whether or not the exponent of force or threats is correct in estimating his rights, he is resorting to extra-judicial means in order to protect a property interest” (Note, A Rationale of the Law of Aggravated Theft, 54 Colum L Rev 84, 98 [1954]). Since such forcible conduct is not merely a transgression against property, but also entails the risk of physical or mental injury to individuals, it should be subjected to criminal sanctions. Consequently, we find the courts in both People v Reid and People v Riddles correctly denied defendants’ requests to assert claim of right defenses.

… We have considered defendant Reid’s remaining points and find them either unpreserved or without merit.

Accordingly, the orders of the Appellate Division should be affirmed.


Notes and questions on People v. Reid

  1. Recall that in People v. Williams, the California case presented early in this chapter, the court relied on common law property crime definitions (including larceny) to interpret the state robbery statute. Here in Reid, the New York court takes a similar approach, perhaps with even better statutory authority: the New York statute at issue here explicitly refers to larceny to define the crime of robbery. That approach – understanding robbery as larceny + the use or threat of force – creates the legal question that arises in Reid: should a circumstance that provides a defense to larceny also provide a defense to the more serious crime of robbery?
  2. To think through that question, it may help to put the use of force aside for a moment and focus solely on larceny. Why is a good faith claim of right a defense to the crime of larceny?
  3. Do the considerations that make a good faith claim of right relevant to liability for larceny also apply in the robbery context? Why or why not?
  4. In 2007, the former football star O.J. Simpson was arrested after he and five other men confronted sports memorabilia dealers in a Las Vegas Hotel and took items at gunpoint. Simpson later claimed that he was simply recovering commemorative items and awards that had been stolen from him. Would Simpson’s claim, if true, exonerate him from a charge of robbery under the analysis of State v. Reid? For the analysis of a Nevada court (applying a Nevada robbery statute), see Simpson v. State, 367 P.3d 819 (Nev. 2010).

Arson

Md. Crim. L. § 3-204. Reckless endangerment [as of 2001]

(a) A person may not recklessly:

(1) engage in conduct that creates a substantial risk of death or serious physical injury to another; or

(2) discharge a firearm from a motor vehicle in a manner that creates a substantial risk of death or serious physical injury to another.

Md. Crim. L. § 6-102. Arson [as of 2001]

(a) A person may not willfully and maliciously set fire to or burn:

(1) a dwelling; or

(2) a structure in or on which an individual who is not a participant is present.


Reginald T. HOLBROOK

v.

STATE of Maryland

Court of Appeals of Maryland
772 A.2d 1240

June 5, 2001

HARRELL, Judge.

Following a non-jury trial in the Circuit Court for Wicomico County, Reginald T. Holbrook (Petitioner) was convicted of first degree arson, eight counts of reckless endangerment, and making a threat of arson… We granted Petitioner’s writ of certiorari…

I.

… There is no significant dispute about the facts in this case. In 1998, Alisha Collins leased a residence at 230 Ohio Avenue in Salisbury, Maryland. Between April and May of that year, nine people lived there [including] Alisha Collins, … her aunt, DeKota Collins, … and, Mr. Holbrook, who was DeKota Collins’s boyfriend. Mr. Holbrook resided at the home for several months and contributed to the rent.

DeKota Collins was the representative payee for Mr. Holbrook’s social security payments. On May 1, 1998, Mr. Holbrook and DeKota Collins had an argument over his money during which he made a menacing gesture toward her with a screwdriver. Alisha Collins called the police. The responding officer told Mr. Holbrook that he would have to leave and not to return to the premises. The officer stayed while Mr. Holbrook removed all of his belongings. Alisha Collins testified at trial that Mr. Holbrook was “really mad.”

About an hour after leaving the premises, Mr. Holbrook returned and asked to speak to DeKota. She told him, “Reggie, I don’t want you no more. I just want you to leave me alone and don’t come back here no more.” Mr. Holbrook sat on the porch and cried. About one hour later, Alisha Collins and her husband left the premises with Mr. Holbrook. The three shared a cab ride, during which Mr. Holbrook repeatedly said “I’m going to get all of you.”

On May 6, 1998, Alisha Collins observed Mr. Holbrook walking back and forth across the street from her house. She testified that he said “I’ll burn this mother fucker up.” Over the objection of defense counsel, Alisha Collins testified that a week before Mr. Holbrook left the home, she overheard an argument between him and DeKota Collins during which Mr. Holbrook said “I’ll burn this mother fucker house down” and “I got people that can hurt you that live upstate.”

[Quoted from the lower court opinion:] On the evening of May 7, 1998, Mr. Holbrook came to the door of the home and asked to see DeKota Collins. Alisha Collins lied and said that she was not home. Mr. Holbrook remained outside of the house for about 45 minutes calling DeKota’s name… That night, Alisha Collins fell asleep on the living room sofa. Sometime after midnight, she awoke to the smell of smoke. She awoke her husband, who went out the back door and discovered a pillow burning on the back porch. All of the occupants safely evacuated the house.

[Also from lower court opinion:] Kevin Ward, a firefighter with the Salisbury Fire Department, testified that the flames from the burning pillow were about 6 to 12 inches high when he arrived, and that there were char marks on the threshold to the rear door and smoke in the basement. Alisha Collins testified that she saw Mr. Holbrook across the street 10 to 15 minutes after the fire was discovered. She told the police that Mr. Holbrook started the fire. Mr. Holbrook was questioned by the police and by the fire marshal. He was subsequently arrested and charged with arson, reckless endangerment, and threats of arson.

On 29 April 1999, Petitioner was tried in a bench trial in the Circuit Court for Wicomico County. The court found Petitioner guilty of one count of first degree arson, eight counts of reckless endangerment [one count for each of the eight persons present in the house at the time of the fire], and one count of making a threat of arson. [The threat charge is not before this Court.] At the 28 June 1999 sentencing proceeding, defense counsel requested that the trial judge merge the reckless endangerment convictions into the first degree arson conviction; the court declined. Petitioner received a 30 year sentence for the arson conviction, with all but 22 ½ years suspended. For the first reckless endangerment conviction, Petitioner was sentenced to five years, to run consecutive to the arson sentence. For each of the remaining seven convictions of reckless endangerment, Petitioner received five years, to run consecutive to the arson sentence, but concurrent to the first reckless endangerment sentence, as well as to each other.

… Petitioner contends that the Court of Special Appeals erred in holding that a conviction and consecutive sentence for reckless endangerment did not merge into the conviction and sentence for first degree arson, when the reckless endangerment was the creation of risk of harm to persons inside a dwelling where Petitioner set a fire on a porch, and the first degree arson was the setting of the fire at the dwelling.

II.

Petitioner argues that, under either the required evidence test [for violations of the Double Jeopardy Clause] or the rule of lenity, or for reasons of “fundamental fairness,” the reckless endangerment convictions and sentences should have merged into the arson conviction and sentence. Concluding that arson and reckless endangerment are separate and distinct crimes, we disagree with Petitioner’s assertion. For reasons we shall explain, we hold that, under the circumstances of this case, the Court of Special Appeals did not err when it affirmed the Circuit Court’s refusal to merge reckless endangerment with arson.

III.

We reiterate that “the cardinal rule of statutory interpretation is to ascertain and effectuate the intention of the legislature.” When striving to determine the legislative intent of any statute, we first examine the plain language of the statute.

Ordinarily, we afford the words of the statute their natural and usual meaning in the context of the Legislature’s purpose and objective in enacting the statute. Moreover, we should avoid “resorting to subtle or forced interpretations for the purpose of extending or limiting [the statute’s] operation.”

A. Common Law and Legislative History

1. Reckless Endangerment

Reckless endangerment is purely a statutory crime. Modeled after § 211.2 of the Model Penal Code and first enacted in Maryland [in] 1989, reckless endangerment was codified originally … under the subtitle destroying, Injuring, etc., Property Maliciously. Effective 1 October 1996, the Legislature repealed [the first reckless endangerment statute] … enacting in its stead Md. Code Art. 27, § 12A–2 under the subtitle of assault. This statute presently provides, in pertinent part:

(a) Creation of substantial risk of death or serious physical injury; penalties.

—(1) Any person who recklessly engages in conduct that creates a substantial risk of death or serious physical injury to another person is guilty of the misdemeanor of reckless endangerment and on conviction is subject to a fine of not more than $5,000 or imprisonment for not more than 5 years or both.

* * * *

(c) More than one person endangered.—If more than one person is endangered by the conduct of the defendant, a separate charge may be brought for each person endangered.

 … In two recent cases, we have discussed the legislative underpinnings of the reckless endangerment statute, as well as the elements of the crime. In State v. Pagotto (2000), we noted that

[t]his statute is aimed at deterring the commission of potentially harmful conduct before an injury or death occurs. The statute was enacted “to punish, as criminal, reckless conduct which created a substantial risk of death or serious physical injury to another person. It is the reckless conduct and not the harm caused by the conduct, if any, which the statute was intended to criminalize.” Thus, the focus is on the conduct of the accused.

… In Jones v. State (2000), we concluded that

[t]he elements of a prima facie case of reckless endangerment are: 1) that the defendant engaged in conduct that created a substantial risk of death or serious physical injury to another; 2) that a reasonable person would not have engaged in that conduct; and 3) that the defendant acted recklessly.

Noting that most Maryland cases addressing these elements discuss the requisite mental state to sustain a reckless endangerment conviction, both Pagotto and Jones cite to Minor v. State (1992), where the Court adopted and applied an objective mens rea:

[G]uilt under the statute does not depend upon whether the accused intended that his reckless conduct create a substantial risk of death or serious injury to another. The test is whether the appellant’s misconduct, viewed objectively, was so reckless as to constitute a gross departure from the standard of conduct that a law-abiding person would observe, and thereby create the substantial risk that the statute was designed to punish.

2. Arson

At common law, arson was defined as the malicious burning of the dwelling of another. Moreover, “at common law, arson [was] an offense against the security of habitation or occupancy, rather than against ownership or property.”

To be convicted of common law arson, the State had to establish four elements: (1) that the building burned was a dwelling house or outbuilding within the curtilage; (2) that the building burned was occupied by another; (3) that the building was actually burned, as mere scorching would not suffice; and, (4) that the accused’s mens rea was willful and malicious.

…The present day arson statute, under which Petitioner was convicted, defines arson as “willfully and maliciously set[ting] fire to or burn[ing] a dwelling or occupied structure, whether the property of the person or another.” Md.Code Art. 27, § 6(a). “Dwelling,” the term applicable in this case, is defined as “a structure, regardless of whether an individual is actually present, any portion of which has been adapted for overnight accommodation of individuals.” Md.Code Art. 27, § 5(b). Additionally, “maliciously” is defined as “ an act done with intent to harm a person or property,” Md.Code Art. 27, § 5(c), while “willfully” is defined as “an act which is done intentionally, knowingly, and purposefully.” Md.Code. Art. 27, § 5(f).

IV.

A. Required Evidence Test

Under the Double Jeopardy Clause of the Fifth Amendment of the United States Constitution, the State can neither hold multiple trials nor punish a defendant multiple times for the same offense. Where a legislature, however, specifically authorizes cumulative punishment under two statutes irrespective of whether they prohibit the same conduct, such punishment may be imposed under the statutes in a single trial. Jones (“[T]he Double Jeopardy Clause does no more than prevent the sentencing court from proscribing greater punishment than the legislature intended.”)).

In the present case, Petitioner received multiple punishments for the same conduct under two statutes in a single trial. As the Court of Special Appeals noted correctly, under Maryland common law, the required evidence test is the appropriate “test for determining whether the different statutory or common law offenses, growing out of the same transaction, are to merge and be treated as the same offense for double jeopardy purposes.” [In a footnote, the court observed that “the required evidence test is commonly referred to as the Blockburger test, see Blockburger v. United States (1932),” and is also sometimes called “the same evidence test,” the “elements” test, or the “same elements” test.] The required evidence test

is that which is minimally necessary to secure a conviction for each … offense. If each offense requires proof of a fact that the other does not, or in other words, if each offense contains an element which the other does not, the offenses are not the same for double jeopardy [and merger] purposes, even though arising from the same conduct or episode. But, where only one offense requires proof of an additional fact, so that all elements of one offense are present in the other, the offenses are deemed to be the same for double jeopardy [and merger] purposes.

As a matter of course, merger occurs when two offenses are based on the same act or acts and are deemed to be the same under the required evidence test; however, “the Legislature may punish certain conduct more severely if particular aggravating circumstances are present … by imposing punishment under two statutory offenses.”

In Petitioner’s view, “every first degree arson necessarily involve[s] a reckless endangerment,” but not vice versa. This assertion, however, is anomalous in light of the language of the statutes. Instead, we agree with the State’s and the Court of Special Appeal’s positions that arson and reckless endangerment do not merge under the required evidence test because each offense has an element not present in the other.

As discussed supra, the offense of arson requires a defendant to act “willfully and maliciously,” while the reckless endangerment offense requires proof that the defendant acted “so reckless[ly] as to constitute a gross departure from the standard of conduct that a law-abiding person would observe.” Petitioner argues that these mens reae are one and the same… While this reasoning may have been true [under prior versions of our statutes, it is not true today]. Section 5 of Art. 27, which provides the definitions for the terms used within the arson statute, defines “maliciously” as “an act done with intent to harm a person or property,” and “willfully” as “an act which is done intentionally, knowingly, and purposefully.” Examining the plain language used to define “maliciously” and “wilfully,” we conclude that the Legislature intended for arson to be a specific intent crime.

Conversely, the Legislature clearly intended for reckless endangerment to be a general intent crime, one whose mens rea requirement is the conscious disregard of the risks and indifference to the consequences to other persons…

We distinguish further the elements of these offenses, for, in contrast with reckless endangerment, arson clearly is defined as a crime against habitation. To reiterate, Art. 27, § 6 provides that “[a] person may not wilfully and maliciously set fire to or burn a dwelling or occupied structure, whether the property of the person or another.” In the present case, the record reflects that, the day before the incident, Petitioner threatened to “burn this mother fucker up” and to “burn this mother fucker house down.” Applying the statute to this evidence, the Circuit Court convicted Petitioner of wilfully and maliciously setting fire to or burning the Collinses’ dwelling. Because dwelling “means a structure, regardless or whether an individual is actually present, any portion of which has been adapted for overnight accommodation of individuals” (emphasis added), we conclude that, in keeping with its common law roots, first degree arson is a crime against habitation, not persons or property. In contrast, reckless endangerment, in keeping with its statutory construction, is a crime against persons, not habitation or property. This is indicative, though not dispositive, of a legislative intent that the crimes may be punished separately. This bears on our later analysis of the rule of lenity with greater weight.

…We reject Petitioner’s argument that, under the required evidence test, the same evidence necessary to convict on the arson offense would always be sufficient to establish the reckless endangerment offense. Accordingly, Petitioner was not convicted twice for the same offense in violation of the Double Jeopardy Clause of the United States Constitution.

B. The Rule of Lenity

When, as in the present case, two offenses do not merge under the required evidence test, we nonetheless may consider, as a principle of statutory construction, the rule of lenity, which “provides that doubt or ambiguity as to whether the legislature intended that there be multiple punishments for the same act or transaction will be resolved against turning a single transaction into multiple offenses.”

… We believe that the Legislature moved the offense of reckless endangerment to its current subtitle in an effort to avoid the very guesswork that Petitioner encourages us to engage in today: whether reckless endangerment could be a crime against property or habitation as well as against persons. We note that, like attempt to commit a crime, reckless endangerment is an inchoate crime, for it “is intended to deal with the situation in which a victim is put at substantial risk of death or serious bodily harm but may, through a stroke of good fortune, be spared the consummated harm itself.” In this case, Petitioner was convicted of recklessly endangering the Collins family by setting fire to a pillow on their porch even though, through a stroke of good fortune, he caused no injury to them. But what if Petitioner had intended to harm the Collinses, and he in fact did cause such harm? What if his crime was no longer inchoate, but complete? It is our view that, even if Petitioner’s intent was not general, but specific as to harming the Collins family, and even if the act of burning the pillow had caused an injury to one or more of the Collinses, the completion of the mens rea and the actus reus would not have ripened into the offense of arson, but rather into the offense of battery, or worse. It, however, would not have ripened under the rule of lenity into the offense of arson.

We believe that there is clear legislative intent that persons convicted of arson also may be convicted of reckless endangerment. It is not logical to assume that the Legislature intended that reckless endangerment would merge for purposes of sentencing with arson. Rather, the General Assembly intended arson and reckless endangerment to be separate offenses subject to multiple punishments. Because there is no doubt or ambiguity as to whether the Legislature intended that there be multiple punishments for Petitioner’s act, the punishments are permitted and the statutory offenses do not merge for sentencing purposes.


Notes and questions on Holbrook v. State

  1. Holbrook involves a multiple-charges situation somewhat similar to State v. Begaye, discussed earlier in this chapter. Recall that in Begaye, the defendant was charged with two separate offenses, burglary and breaking and entering, for the same conduct. The Begaye court rejected the defendant’s argument that these separate convictions violated his right to be free from double jeopardy. Here in Holbrook, the defendant argued that he could not be convicted of both arson and reckless endangerment for the same conduct. Does the Maryland court in Holbrook use the same test to assess the double jeopardy claim that the New Mexico court used in Begaye?
  2. Double jeopardy claims are just one of many contexts in which it is important to be able to identify the separate elements of an offense, including any mens rea requirement. If you were to list the separate elements of the Maryland reckless endangerment offense, what elements would you include? What is the mens rea requirement of that statute?
  3. The Holbrook court says that arson is an offense against habitation, rather than an offense against property. What might be the difference?
  4. Some common law courts held that it was impossible to commit arson against one’s own property. As with burglary, this principle has changed with modern criminal statutes. Many states now explicitly define arson to include intentional destruction by fire of one’s own property. And some states have a specific offense of burning one’s own property in order to collect insurance proceeds.
  5. Is arson a “violent” crime? Again, we will discuss the classification of crimes as violent, and consider “force” as an element of criminal offenses, in more detail in the next chapter. But it is worth noting now that one approach to “violent crime” defines the category to include offenses that involve “the use of force against the person or property of another. Under that definition, arson against someone else’s property could be a violent crime, but arson against one’s own property would not be violent. See, e.g., United States v. Wilder, 834 F. App’x 782, 784 (4th Cir. 2020).
  6. The Maryland arson statute is fairly typical in requiring “willful and malicious” intent; many other arson statutes use similar mens rea language. The terms “malice” and “malicious” appear frequently in criminal statutes, but there is no single uniform definition of these terms. (Nor do courts always agree what “willful” means.) Chapter Six provides greater detail of the term “malice” in the specific context of homicide law. As the Holbrook court notes, the Maryland arson statute defines “maliciously” to mean “with intent to harm a person or property.” Given this definition, the court categorizes arson as a “specific intent crime” and thus distinguishes it from reckless endangerment (a “general intent crime”), allowing the defendant to be convicted of both offenses.
  7. In California, arson is similarly defined with a mens rea of “willfully and maliciously.” “A person is guilty of arson when he or she willfully and maliciously sets fire to or burns or causes to be burned or who aids, counsels, or procures the burning of, any structure, forest land, or property.” Cal. Penal Code § 451. But the California Supreme Court has interpreted this language to create a “general intent” crime that requires only that the defendant intend to engage in the conduct that starts the fire. Under California law, it is not necessary to show the defendant intended to cause any harm, or even intended to start a fire, in order to convict a defendant of arson. The California court reached this conclusion in In re. V.V., 252 P.3d 979 (2011), which involved two minors who had lit a firecracker on a hill in Pasadena. The firecracker exploded and caused a brush fire. The Supreme Court found sufficient evidence of “willful and malicious” intent, emphasizing that the minors had deliberately set off the firecracker, and that a reasonable person would realize that the firecracker could start a fire. It did not matter, the court said, if these defendants did not intend to set a fire, or if these defendants were not actually aware of the risk that the firecracker would start a fire.

    V.V. and J.H. were not required to know or be subjectively aware that the fire would be the probable consequence of their acts. … A defendant may be guilty of arson if he or she acts with awareness of facts that would lead a reasonable person to realize that the direct, natural, and highly probable consequence of igniting and throwing a firecracker into dry brush would be the burning of the hillside….

    Although V.V. and J.H. did not intend to set the hillside on fire, they knew that their intentional acts created a fire hazard. J.H. told the police he attempted to throw the firecracker onto a concrete area on the hillside, while V.V. said they wanted to throw the firecracker onto a green area on the hillside.

    In re V.V., 252 P.3d at 985.

End of Chapter Review

Check Your Understanding (5-3)

Cumulative Review

Check Your Understanding (5-4)

 


  1. [Fn. 3 of original opinion]. As to the definition of “possession” the judge instructed the jury as follows:

    • The word “possession” includes actual as well as constructive possession, and also sole as well as joint possession.
    • A person who has direct physical control of something on or around his person is in actual possession of it.
    • A person who is not in actual possession, but who has knowledge of the presence of something and has the authority or right to maintain control of it either alone or together with someone else, is in constructive possession of it.
    • If one person alone has possession of something, possession is sole. If two or more persons share possession, possession is joint.
    I Iowa Criminal Jury Instructions no. 200.47. As we address below, these instructions are more helpful in controlled substance cases. Jury instructions on the definition of “possession” and “control” under § 714.1 should be based upon the concepts articulated in the Model Penal Code.